Космос и астрономия

Ответить в тред Ответить в тред
Check this out!
Тред тупых вопросов N146 Kumar edition Аноним 14/02/21 Вск 10:16:00 6355331
LK-Advani.jpg 16Кб, 650x429
650x429
IMG202102140937[...].jpg 43Кб, 400x265
400x265
IMG202102140950[...].jpg 129Кб, 1322x1016
1322x1016
15735985979320 [...].jpg 4088Кб, 2498x3548
2498x3548
Тред имени большого учёного Шива Кумара. Спасибо тебе, дядь Кумар, что придумал про коричневые карлики, они красивые. Вопросов о жизни, Вселенной и всём таком тред.

Спрашиваем то, за что в других местах выдают путёвку в биореактор. Здесь анонимные учёные мирового уровня критически рассмотрят любые гениальные идеи и нарисованные в Paint схемы.

Предыдущий тут: >>631039 (OP) (OP)
https://2ch.hk/spc/res/631039.html

Q: Можно быстрее?
A: Можно упасть в пузырь Ай-Тойона, NASA уже почти надула его.

Q: Я начитался охуительных историй про уфологию, че делать, нам жопа?
A: Да, тебе жопа, можешь сгонять в зогач или куда оттуда пошлют.

Q: Я слышал Бетельгейзе взорвалась. Мы обречены?
A: Да, анон, уже запущен обратный отчёт. Держим руку на пульсе прямо здесь https://2ch.hk/spc/res/546582.html

Q: Что будет с человеком в вакууме без скафандра / если он упадет на черную дыру / попробует ступить на поверхность газового гиганта/солнца?
A: Он умрёт.

Q: Почему бы не привязать ракету к воздушному шару или стартовать с горы?
A: Роскосмос - это не как высоко, и не как быстро, большая часть энергии ракеты уходит на разгон вбок (в грунт)
Подробнее тут https://what-if.xkcd.com/58/ (английский) https://chtoes.li/orbital-speed/ (перевод)
Аноним 14/02/21 Вск 10:30:28 6355442
Вкатился в легитимный. Слава Кумару!
Аноним 14/02/21 Вск 10:33:04 6355463
О, неплохо, что кто-то перекатывает тред вовремя, а то меня уже заебало.
Аноним 14/02/21 Вск 10:57:29 6355564
Ну и кумар тут у вас.
Аноним 14/02/21 Вск 11:22:17 6355585
14785333994000.webm 18676Кб, 840x354, 00:05:06
840x354
Аноним 14/02/21 Вск 11:45:39 6355656
Аноним 14/02/21 Вск 21:19:44 6356177
Почему вулканическая магма не радиоактивная? Она же из недр Земли выходит.
Аноним 14/02/21 Вск 21:21:18 6356188
Может ли газовый гигант быть приливно захваченным звездой и постоянно быть повёрнутым одной стороной к звезде? Что тогда будет происходить с атмосферой и недрами газового гиганта, там же нет поверхности твёрдой.
Аноним 14/02/21 Вск 21:22:10 6356199
Какие минусы могут быть у сильного магнитного поля? Или же только плюсы? Что если бы у Земли было магнитное поле в 1000 раз сильнее настоящего? Только компасная стрелка сильнее указывала?
Аноним 14/02/21 Вск 21:23:21 63562010
Почему сколько я живу всё время пиздят про солнечные паруса и их эффективность, а до сих пор все летают на вонючке?
Аноним 14/02/21 Вск 21:26:32 63562111
Достаточно ли атмосферы на высоте 100км, чтобы провести ударную волну от взрыва? Допустим где-то на земле взорвались ебические 200 мегатонн, достанет ли ударная волна до спутника на высоте 100км?
Аноним 14/02/21 Вск 21:28:07 63562212
Если закопаться в грунт Венеры, там будет теплее с глубиной или холоднее? Насколько глубоко пиздец от атмосферы прогревает землю на Венере и как сильно там генерируется тепло под землей?
Аноним 14/02/21 Вск 21:33:21 63562313
Если свести астероид с орбиты, можно ли прорыть канал взрывом? Типа вот гора какая-то блокирует дорогу, почему бы не свести с орбиты астероид и просто не сровнять эту гору с землей. Почему никто не додумался до использования астероидов в мирных хозяйственных целях для рытья каналов, котлованов, проходов в горах?
Аноним 14/02/21 Вск 22:27:30 63563214
>>635617
Там много дыма
>>635618
>происходить с атмосферой и недрами
БУДЕТ ДУТЬ! недра будут крутить, они крутят как solid body на глубине 1000+
>>635619
одни плюсы
>>635620
парус слабый, дым реактивный сильный, летают на дыме
>>635621
нет
>>635622
венера как земля, горячо внутри, гарик, уголёк, сверху горячо, холодно - нигде
>>635623
астероид фугасный, хуярит во все стороны, туннель не сделать, гору можно убрать
Аноним 15/02/21 Пнд 08:22:31 63566415
Может ли смартфон на 845 снэпдрегоне заменить бортовой компьютер космического корабля?

Диды летали с оборудованием, в сот раз слабее современного бытового устройства для потребления контента.
Аноним 15/02/21 Пнд 11:06:06 63567016
>>635664
> Может ли смартфон на 845 снэпдрегоне заменить бортовой компьютер космического корабля?
Отказоустойчивость в условиях радиации и низких/высоких температур обеспечь - и дело в шляпе.
Аноним 15/02/21 Пнд 11:13:18 63567117
>>635623
Астероид тягать дороже, чем такую же мощность в тротиловом эквиваленте даже тротилом обеспечить.
Аноним 15/02/21 Пнд 11:29:04 63567318
анончик, анончик, давай выпьем чай подскажи мне расстояния планет от слонышка в среднем, при условии, что мы берём расстояние до меркурия за 1 метр
Аноним 15/02/21 Пнд 11:36:17 63567519
>>635673
Ты про закон Тициуса-Биде хочешь послушать, или глазами посмотреть?
Вот хорошее видео: https://youtu.be/Kj4524AAZdE
Аноним 15/02/21 Пнд 11:54:30 63568020
>>635675
мне сложны такие величины. я же попросил - в метрах
Аноним 15/02/21 Пнд 12:00:13 63568221
>>635680
Тогда планеты слишком миллипиздрические будут, не разглядишь.
Аноним 15/02/21 Пнд 12:13:23 63568322
>>635682
да ты мне тут напиши, а не покажи
Аноним 15/02/21 Пнд 12:18:44 63568423
>>635683
Ты арифметику до сих пор не прошел? Ладно, вот:
Ртуть - 1 м
Венера - 1.84 м
Земля - 2,56 м
Марс - 3,9 м
Церера - 6,9 м
Юпитер - 13,33 м
Сатурн - 24,3 м
Твойанус - 49,23 м
Шептун - 77,2 м
Плутон - 101,38 м
Седна - 1 километр 298 метров
Аноним 15/02/21 Пнд 12:21:03 63568524
>>635684
Облако Оорта - 192 километра
Проксима Центавры - 687 километров
Аноним 15/02/21 Пнд 12:23:59 63568625
>>635685
А ядро нашей голактике?
А тумманость Андромеды?
Аноним 15/02/21 Пнд 12:28:04 63568726
>>635686
4 миллиона 157 тысяч километров.
411 миллионов километров.
Аноним 15/02/21 Пнд 12:45:17 63568827
1. Поясните, почему метеориты входят в атмосферу земли всегда под острым углом?
2. Если бы угол входа челябинского метеорита был не 18 градусов, а 90, то каковы были бы последствия?
Аноним 15/02/21 Пнд 12:48:56 63568928
>>635688
1. Потому что большая часть траекторий не прямо в земляшку.
2. Мощнее жахнуло бы.
Аноним 15/02/21 Пнд 12:50:29 63569029
>>635688
1 потому что земля круглая, для прямого угла нужно чтобы траектория была точно в центр
2 наверное, ударился в землю бы
Аноним 15/02/21 Пнд 12:53:52 63569130
>>635689
>2. Мощнее жахнуло бы.
НУ как бы спасибо капитан очевидность.
Я имею в виду разрушения какие бы были? Примерно так.
Скажем снесло бы хотя бы пару зданий?
Аноним 15/02/21 Пнд 12:56:32 63569231
>>635691
Интересно, что было бы, если Тунгусский метеорит ёбнул бы не в тайгу а прям по Красной площади в ДС..?
Аноним 15/02/21 Пнд 12:57:27 63569332
image.png 158Кб, 440x440
440x440
Аноним 15/02/21 Пнд 14:37:23 63570133
>>635684
>Венера - 1.84 м
>Земля - 2,56 м
>Марс - 3,9 м
до марса в среднем 1,5 м, а до венеры 0,5 м. почему мы всё время летаем зырить, чё там у марса, вместо того, чтобы дудосить свинеру исследовательскими спутниками?
Аноним 15/02/21 Пнд 14:40:36 63570234
>>635701
Потому что на марс можно сесть и жопа от давления и температуры не отвалится
Аноним 15/02/21 Пнд 14:43:58 63570335
>>635701
До туалета 3 метра, а до кухни 7 метров, почему я хожу на кухню есть? Потому, что на толчке воняет, вот почему. Да, мое место у параши.
Так и свинерой, анон верно ответил, там не потусишь, в этом аду ловить попросту нехуй.
Аноним 15/02/21 Пнд 21:39:10 63577136
Выгоднее ли тормозить об атмосферу юпитера, вместо того чтобы тащить химию для двигателей? Или же щит будет тяжелее топлива необходимого для захвата?
Аноним 15/02/21 Пнд 21:46:18 63577237
>>635771
А ты можешь вообще не об атмосферу, сделав гравиманевр у одной из лун.
Аноним 15/02/21 Пнд 22:13:14 63577538
>>635772
Кажется если ганимеда юзать, то всё равно нужно будет пукнуть в какой-то момент, нет?

Алсо, что выгоднее юзать для гравиманёвра - Ио или Ганимед? Ио же плотнее, там гравитация больше, даже если Ганимед по массе сам тяжелее.
Аноним 15/02/21 Пнд 22:56:04 63577939
Вселенная бесконечна или нет?
Аноним 15/02/21 Пнд 23:12:57 63578140
>>635771
Выгоднее
Например, щит марсоходов весит около 300 кг, берет на себя около 5-6 км/с дельты у аппарата в пару тонн
Топлива же нужно в раз 20-30 больше по массе для такого
Аноним 15/02/21 Пнд 23:25:05 63578241
>>635779
> Вселенная бесконечна или нет?
Ты сначала до соседней планеты долети и потопчись на ней ножками, а потом уже о конечности вселенной вопросы задавай, человечищка.
Аноним 15/02/21 Пнд 23:28:45 63578342
>>635779
Сложный вопрос, на который нам как бы рановато давать ответ. Человечество существует всего как бы 2500 лет более менее разумно, а вы хотите узнать уже всё о вселенной. Зачем, я вообще задумываюсь. Как ответ на этот вопрос может изменить поведение человека на микроуровне в его личной жизни например, вот узнает человек что вселенная бесконечна, будет ли он учиться и работать усерднее, или же если он узнает что она конечно, то будет ли он менее серьезно относиться к своей жизни? А что именно подразумевается под конечностью, время или пространство? Прям, зачем людям знать такое, в толк не возьму

мимопришелец
Аноним 15/02/21 Пнд 23:56:35 63578843
>>635783
Просто интересно.
Аноним 16/02/21 Втр 00:18:45 63579044
15f1c53539224e0[...].jpg 465Кб, 1242x932
1242x932
>>635788
Просто этот комикс вспоминается. Мы тут вокруг тебя такие же хомячки-папаши, которые нихуя не знают. Просто люди называющие себя учеными часто притворяются, что у них есть все ответы, потому что круто выглядеть умными, а говорить "Мы не знаем" как-то стрёмно, поэтому они выдвигают гипотезу какую-то и все вокруг притворяются, что это не гипотеза, а уже нечто почти доказанное, чему можно учить как факту и от чего можно отталкиваться в построении новых гипотеза, которые как каскадом опираются на первую гипотезу, а потом когда эта изначальная гипотеза оказывается ложной, ученые внезапно вспоминают, что это была гипотеза, ага, не было никакой ошибки, никто не преподавал её как факт, не притворялся, что на основе этой гипотезы знает природу реальности, нееее, ти чо)))
Аноним 16/02/21 Втр 00:24:30 63579145
>>635533 (OP)
Слышал версию о том, что мы не нашли разумную жизнь за пределами Земли потому что мы одни из первых. А что если наоборот - одни из последних? Ведь пик звездообразования давно пройден, в том числе и высокометалличных звезд, а значит, мы чисто статистически в меньшинстве. Что если мы найдем в космосе не собратьев по разуму, а давно мертвые руины, следы миллиардов войн, межгалактических империй, разваливающиеся мегасооружения и до сих пор летящие по инерции боевые ракеты, выпущенные миллиарды лет назад?
Аноним 16/02/21 Втр 00:28:03 63579246
>>635790
>люди называющие себя учеными часто притворяются, что у них есть все ответы, потому что круто выглядеть умными,
Нет, не поэтому.
Все ответы всегда есть у религиозной и шизотерической шизы.
Сказать, что не знаешь, значит отправлять баранов искать ответов у шизотериков.
Аноним 16/02/21 Втр 00:30:17 63579447
>>635791
ну найдём и найдём, мы никогда не сможем исключить существование разумной жизни где-то на другом конце галактики, просто потому что никогда не сможем до туда добраться. А уж про другие галактики и речи не идёт, если мы никогда не сможем прочесать какую-нибудь галактику треугольник или сомбреро на признаки цивилизаций, то как можно заявлять, что их там нет? Многие широкие заявления о существовании или отсутствии разумной жизни и цивилизаций в даже в этой галактике имеют мало смысла в виду невозможности централизованно знать о состоянии всех звёзд в галактике одновременно чисто из-за ограничения скорости света, но подобные позитивные или негативные заявления о пришельцах и иноцивилизациях вообще теряют смысл, когда мы говорим о нескольких галактиках или шире, галактических скоплениях недоступных для изучения.
Аноним 16/02/21 Втр 00:34:58 63579548
>>635792
Нет, иногда нужно уметь сказать - "мы не знаем" или еще лучше "мы никогда не узнаем", как относительно недр черных дыр и состояния до большого взрыва.
который кстати сам по себе может быть одной из гипотез, которая тоже как временная пробка существует лишь для создания видимости знания , но лучше в эту тему даже не лезть.
Если же спешить с выводами, то можно нараздавать несколько нобелевок, написать тысячи учебников, создать видео, документалки и курсы, которые окажутся бессмысленны, когда окажется что гипотеза на основе которой всё это делалось была ошибочна. Такое может произойти, когда если Вебб начнёт пырить в небо и не увидит следов никакой тёмной энергии. Такой просак может навредить авторитету науки куда хуже, чем если бы немного повременили бы с далекоидущими выводами.
Аноним 16/02/21 Втр 00:38:07 63579649
>>635795
Наука не должна выяснять истину. Она лишь помогает создавать картину мира, которая с приемлемой точностью согласуется с реальностью и нужна для предсказания этой реальности. Создал теорию, которая на писечку точнее предыдущей и охватывает чуть больше случаев, устраняя два закона, сводя все в один? Молодец, держи нобелевку.
Аноним 16/02/21 Втр 00:47:16 63579750
>>635796
та не пизди, в космологии видать нобелевки раздают без такой вот долгой философской проверки, а чисто "нихья се ОТКРЫТИЕ, ебать это что-то новенькое, никогда раньше не видели, this could be HUGE if true, держи нобелевку, это революция, РЕВОЛЮЦИЯ ааааа я кончаю, кончаю, никогда такого не было и вот опять!!"
Аноним 16/02/21 Втр 01:32:27 63580251
>>635797
Какой-то ты натужный.
Аноним 16/02/21 Втр 01:37:24 63580452
>>635796
Удваиваю этого постпозитивиста. Наука даже не претендует на то, что она знает какую-то истину, у неё просто есть модели, которые расходятся с реальностью не более чем на некоторую величину.
Аноним 16/02/21 Втр 01:41:45 63580553
>>635795
но ведь в сериале "Звёздные врата: Вселенная", было предположение что реликтовое излучение искусственного происхождения
Мало того, очень высокоразвитая цивилизация даже послала невероятно продвинутый корабль "Судьба" выяснить этот вопрос
Однако сериал прервался на середине, да и создатели Судьбы настолько развились за время долгого полёта, что вознеслись превратившись в энергетический сгусток и очевидно уже в подобном состоянии - получили интересующий вопрос

>Но во втором сезоне сериала выясняется, что главная миссия — изучить реликтовое излучение, которое, как оказалось, содержит в себе сложно структурированный сигнал, возможно, носит искусственный характер и скрывает тайну создания Вселенной. Однако, начав эксперимент миллионы лет назад, древние так и не довели его до конца из-за своего вознесения.
Вот интересно всё таки, что они имели ввиду? Что нам могли показать, если-бы сериал продолжился?
Аноним 16/02/21 Втр 05:44:04 63581554
>>635533 (OP)
Бля позор ...
Во-первых, Алькубье.
Во-вторых, не упасть, а построить вокруг корабля изнутри (иначе не было бы родственников).
В-третьих, НАСА наклоняет десятую часть, контролируя ее с помощью сверхточного интерферометра, прямо здесь, чтобы насмехаться над раком перед Антарктидой.
Аноним 16/02/21 Втр 06:03:00 63581655
>>635790
>>635795
Это не научный подход. Ты, блядь, науку по желтушным публикациям знаешь только что ли? Постоянно говорят "мы не знаем" и это норма в науке, тот анон верно говорит - только религиобляди и прочая шиза может пиздеть со стопроцентной уверенностью.
Ну и ты к ней относишься, раз так уверенно кукарекаешь о том о чем не знаешь.

>>635795
>Если же спешить с выводами, то можно нараздавать несколько нобелевок, написать тысячи учебников, создать видео, документалки и курсы, которые окажутся бессмысленны, когда окажется что гипотеза на основе которой всё это делалось была ошибочна.
Так не происходит, еблан.

>>635797
Нихуя подобного.
Аноним 16/02/21 Втр 06:35:55 63581856
>>635791
>Ведь пик звездообразования давно пройден, в том числе и высокометалличных звезд, а значит, мы чисто статистически в меньшинстве.
Там такое "статистически" что плюс-минус десяток миллионов лет вообще погоды не делают. Так что либо жизнь должна быть очень редкой, либо нам должно очень повезти. 0,1% разницы в возрасте звездных систем и при прочих равных мы/они начинали осваивать космос, пока их/наши обезьяноподобные предки еще камни колоть не умели.
Плюс формирование звезд сильно неравномерно. Скажем, что бы далеко не ходить - Альфа Центавра и старше на ~1.5 миллиарда лет, и металличнее при этом. И это даже выискивать не пришлось, это тупо ближайшая солнцеподобная звезда.
Аноним 16/02/21 Втр 07:38:21 63582357
>>635779
Вселенная замкнутая, как поверхность шарика. То есть она обладает конечным объемом, но у нее нет краев, потому что пространство устроено таким образом, что оно замыкается само на себя. Если бы Вселенная не расширялась с ускорением, в ней можно было бы совершить кругосветное путешествие.
Аноним 16/02/21 Втр 07:39:01 63582458
Аноним 16/02/21 Втр 07:53:46 63582659
>>635823
Пруфай, что у вселенной положительная кривизна.
Аноним 16/02/21 Втр 08:06:13 63582960
https://ru.wikipedia.org/wiki/Корабль_поколений
Возможно ли создание корабля поколений сейчас, на существующей технологической базе?
В качестве двигателя можно взять термоядерный реактор, скажем.
Аноним 16/02/21 Втр 08:08:47 63583061
>>635829
>на существующей технологической базе
>термоядерный реактор
Выбери что-то одно.
Аноним 16/02/21 Втр 11:14:10 63586362
Реально ли обычному хую посмотреть на запуск ракеты в космос?
Аноним 16/02/21 Втр 11:14:57 63586463
>>635863
Смотря с какого расстояния.
Аноним 16/02/21 Втр 11:16:53 63586764
>>635864
А на какое минимальное расстояние пускают мимохуев и пускают ли? Может есть какие-нибудь конторы, организующие экскурсии?
Я просто вообще не в курсе.
Аноним 16/02/21 Втр 11:19:53 63586865
>>635867
На мысе Канаверал с нескольких километров можно спокойно смотреть, глянь те же съемки, когда ФЛАКОН ТЯЖЕЛО летал в первый раз - толпа народу. В РФ хер кто тебя ближе 15-20 км к космодрому просто так пустит, но есть экскурсии на Байконур, там можно так же с нескольких километров посмотреть.
Аноним 16/02/21 Втр 11:20:36 63586966
>>635863
Видео есть жи, значит с некоторого расстояния можно.
Аноним 16/02/21 Втр 11:23:02 63587067
>>635868
>На мысе Канаверал с нескольких километров можно спокойно смотреть, глянь те же съемки, когда ФЛАКОН ТЯЖЕЛО летал в первый раз - толпа народу.
А пидорах туда пускают или только граждан сша?
>но есть экскурсии на Байконур, там можно так же с нескольких километров посмотреть.
Вот здесь поподробнее. Кто, когда, дорого ли.
Алсо байконур это же в казахстане, да?
Аноним 16/02/21 Втр 11:27:05 63587268
>>635870
> пидорах или граждан сша
Чел, ты... Это давно уже одно и то же.
Аноним 16/02/21 Втр 11:29:07 63587369
>>635870
>Кто, когда, дорого ли
В гугле ж не забанили, поищи, по кнопочкам то тыкать умеешь. Да, казахстан. По поводу канаверала не скажу точно, но не думаю, что там полузакрытая зона только для граждан, по крайней мере, точно знаю, что есть те, кто специально летал посмотреть пуски по туристической визе.
Аноним 16/02/21 Втр 11:36:06 63587570
>>635870
Помимо Байконура и Канаверал есть ещё и Куру.
Аноним 16/02/21 Втр 11:39:48 63587671
image.png 230Кб, 1980x1317
1980x1317
>>635867
>Может есть какие-нибудь конторы, организующие экскурсии?
Разумеется.
https://kosmodrom.space
https://baikonurtour.ru/price.html
Их сейчас несколько вообще стало, я даже не знаю, какой валидный, раньше вроде один оператор был.
Аноним 16/02/21 Втр 12:14:15 63588272
Если мы возмём планету массой с Землю и формой с Землю, а потом с массой Земли и с формой баскетбольного мяча, то есть пустую внутри, а, следовательно, большую в объёме, как повлияет на вес, на g, на магнитное поле и т.д.?
Аноним 16/02/21 Втр 12:21:53 63588573
>>635882
Хуй знает, считать надо.
Аноним 16/02/21 Втр 12:22:15 63588774
>>635882
>>635885
Но вообще она схлопнется нахер, а магнитного поля не будет никакого.
Аноним 16/02/21 Втр 12:22:30 63588875
>>635882
Вторая моментально обрушится внутрь себя и получится обычная Земля, только из кипящей лавы.
Аноним 16/02/21 Втр 12:38:03 63589276
>>635887
>>635888
А если у неё каркас и углеродных нанотрубок?
Аноним 16/02/21 Втр 12:42:56 63589477
>>635892
Они на растяжение работают.
Аноним 16/02/21 Втр 13:05:59 63590578
>>635826
При чем тут кривизна? Я говорю о форме, то есть о топологии, а не о геометрии. Наблюдаемой кривизны у Вселенной как раз нет, она плоская.
Аноним 16/02/21 Втр 13:23:12 63591179
>>635905
Как она может быть замкнута, если нет кривизны?
Аноним 16/02/21 Втр 14:20:25 63591680
>>635894
Тогда из углеродных нанофуллеренов...
Аноним 16/02/21 Втр 15:37:45 63592181
>>635911
Ну мы не видим кривизны Вселенной, потому что она очень большая - намного больше видимой ее части. Точно так же мы не видим кривизны Земли, находясь на ее поверхности, поскольку по сравнению с нами Земля просто гигантская и с нашего ракурса кажется плоской.
Аноним 16/02/21 Втр 15:44:07 63592282
А если мощным нюком жахнуть в Олимп, он просрется, или там давно все засохло?
Аноним 16/02/21 Втр 15:46:19 63592383
>>635921
Однако мы можем измерить углы произвольного треугольника и исходя из их суммы прикинуть, в какую сторону кривизна и насколько сильная. И пока все измерения, в пределах погрешностей по крайней мере, показывают, что вселенная таки плоская с огромной вероятностью.
Аноним 16/02/21 Втр 15:53:04 63592484
>>635922
Да хер его знает. Судя по отсутствию магнитного поля у Марса - точно нет активного перемешивания недр, тектоники особо тоже не наблюдается, зафиксированные марсотрясения были с гипоцентром, достаточно близким к поверхности, так что Марс скорее тектонически мертв.
Аноним 16/02/21 Втр 15:56:08 63592585
>>635924
А его недра не прозвонили что ли? На луну вроде сейсмографов набросали.
Аноним 16/02/21 Втр 16:01:23 63592786
>>635925
На Викингах были сейсмографы, но их ветром perdolilo отчего верить им сложно. Из новых на Инсайте есть сейсмометр. Но он только регистрирует, чтобы "прозвонить" надо везти хотя бы три одинаковых сейсмометра в достаточно отдаленные точки планеты и заряды взрывчатки достаточно мощные. И если первое еще можно исполнить, то запустить йобу на пару тон тротилового эквивалента в космос никто не даст, потому что договор о неразмещении оружия в космосе.
Аноним 16/02/21 Втр 16:03:21 63592887
>>635927
А если разгонной ступенью ебануть о планету? Или она сгорит?
Аноним 16/02/21 Втр 16:06:11 63592988
>>635928
Будет даже слабее, чем от падающих метеоритов. По хорошему вообще чем-нибудь килотонным ебнуть надо, чтобы точность не была плюс-минус четверть радиуса планеты. На Земле то естественным источником сейсмических волн служат достаточно мощные землетрясения, энергия которых на порядки выше энергии удара метеорита или разгонной ступени. Можно еще ядерным взрывом, конечно, но такое запустить точно никто не позволит.
Аноним 16/02/21 Втр 16:35:59 63593489
Почему джейм уэб летит на ариадне а не на протоне ?
Аноним 16/02/21 Втр 16:37:35 63593590
>>635934
Сколько аварий у протона?
Аноним 16/02/21 Втр 16:39:17 63593691
>>635934
может себе позволить без шуток
Аноним 16/02/21 Втр 17:33:27 63595192
>>635923
А я о чем говорю? Мы не видим кривизну Вселенной, потому что она намного больше видимой ее части. Видимая же часть выглядит совершенно плоской, подобно тому как поверхность Земли выглядит для нас плоской. Ты, видимо, забыл, что для наблюдений нам доступна лишь часть Вселенной, а не вся Вселенная. Из того, что видимая часть выглядит плоской, мы и делаем вывод, что вся Вселенная намного больше этой части - именно поэтому мы и не видим ее кривизны.
Аноним 16/02/21 Втр 17:37:41 63595293
>>635951
А с какого перепуга ты утверждаешь, что она искривлена? Нужно доказать то, что она искривлена. А доказательств нет.
Аноним 16/02/21 Втр 18:01:57 63595994
>>635823
считается что Вселенная в форме тора

>>635951
Ты ещё не забывая про Мультивселенную или Метавселеную
Даже если наша обозримая Вселенная замкнутая и в форме тора, шара или бутылки Кляйна, это вовсе не исключает бесконечное множество других Вселенных


Аноним 16/02/21 Втр 18:08:21 63596095
>>635868
В турах на байк еще на вывоз ракеты дают посмотреть, там натурально несколько метров. С минимального расстояния даже боковушка в кадр не влезет.
Аноним 16/02/21 Втр 18:09:27 63596196
image.png 115Кб, 630x400
630x400
>>635959
>считается что Вселенная в форме тора
И где мы на нем находимся?
Аноним 16/02/21 Втр 18:09:36 63596297
>>635816
Ну да, это я еблан, а хуесосы сверху рассуждающие о форме вселенной с гордыми умными еблами само воплощение науки. Вселенная в форме бублика, истинно говорю вам огромный анус из большого взрыва вышел и мы в жопе
Аноним 16/02/21 Втр 18:10:21 63596398
>>635962
Да, ты и есть еблан. По паре двачеров и желтушным насильникам о науке судить, нихуя не знаешь - так помалкивай.
Аноним 16/02/21 Втр 18:17:09 63596499
>>635963
Я и реальных ученых с умными ёблами видел, которые такую хуйню задвигали, рядом сидели другие умные ученые и они тупо как родственники, которые живут с шизом и привыкли к нему, не прерывали его, а позволяли ему хуйню нести. Ученые с шизами срослись, ты там границу четко не проведешь, они все тупо толерантные к хуйне и несут её. Всякие митио каки блядь на тв выступают и несут такую хуйню про кротовые норы ведущие в будущее, другие дауны нажрутся кислоты и начинают нести про квантовое сознание и связь квантовых компьютеров с пришельцами, которые упарываются веществами, четвёртые начнут про мультивселенную из пузырей нести, про белые дыры, про хуйню малофью что струны это творение еврейского бога, что они нашли код кабалы в природе вселенной и что наш мир это симуляция Гига-Еврея.
Аноним 16/02/21 Втр 18:20:11 635965100
>>635964
Я не собираюсь тебя читать, просто скрою сразу.
Аноним 16/02/21 Втр 18:48:50 635966101
Аноним 16/02/21 Втр 19:04:42 635968102
image.png 763Кб, 959x729
959x729
Есть ли какие-то объективные причины пикрелейтеда?
Аноним 16/02/21 Втр 19:15:40 635969103
Аноним 16/02/21 Втр 19:42:27 635970104
>>635968
У руководителя НАСА есть приусадебное хозяйство, поэтому деньги ему нужны только на коммунальные платежи.
Аноним 16/02/21 Втр 20:52:24 635974105
Почему на Марс не посылают ниггеров в один конец для строительства баз?
Аноним 16/02/21 Втр 21:00:28 635975106
>>635974
>строительства баз
или в кфц раз? Все выбирают второе.
Аноним 16/02/21 Втр 21:29:25 635977107
>>635975
Не понял тебя, шизик. Но на всякий случай осуждаю за расистские стереотипы.
Аноним 16/02/21 Втр 21:42:43 635980108
image.png 197Кб, 267x290
267x290
Аноним 16/02/21 Втр 21:46:25 635982109
>>635977
>осуждаю з
твитч блядь, спок
Аноним 16/02/21 Втр 21:47:01 635983110
>>635968
потому что насасами печатают доллары и платить могут хоть сколько
Аноним 17/02/21 Срд 00:10:51 635988111
Аноним 17/02/21 Срд 00:12:38 635989112
>>635964
митио каку долбаеб соглы, on par with ne'Gr Tyson
Аноним 17/02/21 Срд 00:26:33 635991113
Анонии, сегодня прогресс вывели 2.5 тонны на орбиту.
А если таким же макаром вывести на орбиту 2.5 тонн радиационных отходов, а потом их сжечь в атмосфере? Радиация же сгорит?


А почему нельзя на луну сбрасывать отходы? Хотя бы по 500 кило за запуск, всяко дешевле чем хранить, нет?
Аноним 17/02/21 Срд 00:33:49 635993114
>>635991
Радиация-то сгорит, а жопы вместе с ней. Общество ещё не готово к прогрессивным методам утилизации. К тому же много денег на этом не "освоишь".
Аноним 17/02/21 Срд 01:09:25 635994115
>>635991
>А если таким же макаром вывести на орбиту 2.5 тонн радиационных отходов, а потом их сжечь в атмосфере?
Нахуя, блядь? Ценнейший источник редких элементов. Алсо, из них даже без плутония можно дохуя энергии намайнить, батарейки всякие делать. Цезий со стронцием чистыми электронами срут, а гамма-квант может сделать вторичные. Это всё техника завтрашнего дня, атомные батарейки как в фоллауте.

>Радиация же сгорит?
А вода сдетонирует. Как у тебя ядра сгорят-то? Они ядра, блядь. Им вообще похуй, что в электронных оболочках делается.

>А почему нельзя на луну сбрасывать отходы?
Потому что нельзя жопу вытирать сусальным золотом, а гвозди забивать микроскопом. Выкидывать ядерные отходы в космос (и тем более на Луну) придумал какой-то феерический мудак-радиофоб, и по своей идиотичности эта идея - это нечто на уровне китайского сверхпроекта по истреблению воробьёв.

>Хотя бы по 500 кило за запуск, всяко дешевле чем хранить, нет?
Нет. Хули там хранить-то? Вкопал в землю бетониума, положил туда контейнеры с отходами, снаружи поставил сторожа с берданкой, всё.
Аноним 17/02/21 Срд 03:37:44 636004116
>>635964
Забыл Тегмарка с его математической вселенной.
Аноним 17/02/21 Срд 06:09:38 636009117
>>635991
>Анонии, сегодня прогресс вывели 2.5 тонны на орбиту.
За ~70 миллионов долларов. Но это с учетом самого Прогресса, если просто на орбиту Союзом, то ~7-8 тонн за ~35 лямов.
>Радиация же сгорит?
Нет, наоборот рассется по большой территории.
>А почему нельзя на луну сбрасывать отходы? Хотя бы по 500 кило за запуск, всяко дешевле чем хранить, нет?
Нет, дороже. Даже просто на орбиту самой дешевой ракетой будет около трех миллионов долларов за тонну. К луне обойдется в разы дороже, начиная лямов от десяти.
Это если по цене кирпичей запускать, на деле будет еще дороже, потому что с самими отходами сложно работать.
Аноним 17/02/21 Срд 08:20:31 636022118
>>635974
Потому что на марс летать ДОРАХА, до сих пор ни одного огурца послать не сподобились, а что для негров, что для профессиональных акробатов астронавтов надо по большей части дохуя в плане жизнеобеспечения. Проблема не в том что астронавты требуют там мазерати припаркованную у входа, а то что все что требуется в целом дохуя стоит.
Аноним 17/02/21 Срд 08:22:27 636023119
>>635991
Если так хочется от ядерных отходов избавляться, то надо реакторы на быстрых нейтронах строить и в них "сжигать" их. Не больно-то и хочется от отходов избавляться, значит.
Аноним 17/02/21 Срд 09:29:03 636032120
Если у магнетара или миллисекундного пульсара скорость вращения на экваторе околосветовая, то получается, что время для такой нейтронной звезды протекает совсем иначе, чем для большей части Вселенной? Получается ли, что в своём времени пульсары замедляются быстро, а в нашем - долго?
Аноним 17/02/21 Срд 10:24:45 636039121
Почему маск в начале 2000х не смог купить ракеты у рф ?
Аноним 17/02/21 Срд 10:30:06 636040122
>>636039
Потому что ему их не продали.
Аноним 17/02/21 Срд 10:31:02 636041123
>>635952
https://m.colta.ru/articles/specials/8165-kak-voznikla-vselennaya-i-chto-s-ney-budet-dalshe
>С переменным успехом теория, которая в 1940-х получила название «теория Большого взрыва», просуществовала до конца 60-х годов. До этого Вселенная была вместилищем всего сущего, но после вмешательства Фридмана, Эйнштейна, Леметра и Хаббла она свой статус потеряла и превратилась в физический объект с разными характеристиками: размер, плотность, температура, свет. А как представить себе этот физический объект? И вот здесь многие ломаются. Потому что как это — представить замкнутую Вселенную? Я сейчас это объясню, и дальше слушать будет легче. Легко себе представить бесконечную Вселенную, правда? А как себе представить конечную Вселенную? Проще всего, наверное, представить себе шарик, на поверхности которого нарисованы галактики, звезды. Шарик можно надувать — тогда он будет расширяться, и нарисованные галактики будут друг от друга удаляться. Очень важно понимать, что у такого расширения нет центра. Почти всегда, представляя себе Большой взрыв, люди думают, что где-то что-то в какой-то точке взорвалось и расширяется в пустоту. Ничего подобного, нет никакой пустоты. Это именно замкнутое пространство, которое легче нам представить на поверхности шарика, которое все расширяется само по себе. В нем нет пустоты, оно однородно, и в нем нет центра.
>Я уже говорил: Вселенная — это поверхность шарика, только не двумерная, а трехмерная (точнее, даже четырехмерная, так как у нее есть время плюс три пространственные координаты). Когда-то Вселенная была очень маленькой. Какие силы на нее действовали? Или — можно на другой язык перевести — какие силы есть в вакууме? Почти никаких. А на поверхности шарика какие силы могут действовать? Сила поверхностного натяжения? А что будет, если мы подставим силу поверхностного натяжения в теорию гравитации Эйнштейна? На самом деле это очень просто показать, но надо писать простейшее дифференциальное уравнение, а я не буду это делать. Ответ такой: сила поверхностного натяжения будет не сжимать, а со страшной силой расталкивать шарик. Если она сильная, шарик будет раздуваться. «Раздувание» по-английски «inflation», оттуда и термин.
>Собственно, основная часть моего доклада подошла к концу. Остался лишь один вопрос: если Вселенная — это физический объект, то значит ли это, что она одна? Да нет, конечно. Сама постановка вопроса говорит, что замкнутых трехмерных шариков, из которых нельзя выпрыгнуть, может быть очень много.
https://coollib.com/b/428217/read#t3
>Как представить замкнутую конечную вселенную? С самым простым способом автор познакомился на первом курсе Физтеха на лекции Сергея Петровича Капицы. Тот брал воздушный шарик, на котором нарисованы завитки-галактики, подсоединял его к трубке компрессора и открывал вентиль. Шар медленно надувался, а Сергей Петрович, разводя руками, показывал, как галактики разбегаются — чем дальше друг от друга, тем быстрей, как и наблюдал Хаббл. Потом шарик громко лопался, и лектор обводил аудиторию победным взглядом.
>Победный взгляд оправдан: демонстрация снимает глупые вопросы: «Где произошел Большой взрыв?», «Откуда разбегаются галактики?» и «Где у Вселенной край?». Многие воспринимают Большой взрыв по аналогии с обычным взрывом: разлет вещества из некоего эпицентра в пустоту. Смотрите на надуваемый шарик, на его поверхность! Там нет и не было центра разлета. Вообразите, что шарик стал раздуваться от микроскопических размеров — сначала быстро, потом медленней. Большой взрыв и есть начало расширения «шарика» — «замкнутой» Вселенной. Надо лишь добавить, что в этой демонстрации есть третье измерение, откуда мы можем рассматривать шарик. Пример будет точнее, если допустить, что все движения возможны только вдоль поверхности шарика, а третьего, перпендикулярного измерения (в случае реальной Вселенной — четвертого пространственного) нет вообще.
>Коль скоро мы признали, что Вселенная — физический объект, имеет смысл, не откладывая, перечислить основные геометрические и физические характеристики этого объекта.
>Определить размер «шарика», в «поверхности» которого мы живем, мы не можем — он слишком велик и весь не доступен наблюдениям (см. ниже про горизонт). Впрочем, были попытки найти объекты, видимые с противоположных направлений, в предположении, что лучи от них обогнули замкнутую Вселенную с противоположных сторон, как взрывная волна от падения Тунгусского метеорита обогнула земной шар. В таком случае можно было бы примерно оценить размер, но сейчас мы точно знаем, что подобное невозможно — Вселенная слишком велика.
>>635959
>считается что Вселенная в форме тора
Ты скозал?
>Ты ещё не забывая про Мультивселенную или Метавселеную
>Даже если наша обозримая Вселенная замкнутая и в форме тора, шара или бутылки Кляйна, это вовсе не исключает бесконечное множество других Вселенных
А при чем тут вообще мультивселенная? Хуйню спизданул какую-то.
Аноним 17/02/21 Срд 10:40:01 636042124
>>636040
А надо было больше предлагать
Аноним 17/02/21 Срд 11:15:36 636047125
>>636032
С чего ты взял, что магнитар или миллисекундный пульсар на экваторе вращается с околосветовой скоростью?
Аноним 17/02/21 Срд 11:19:31 636048126
>>636047
>магнитар
Пиздуй в свой n+1, уважаемый. М-А-Г-Н-Е-Т-А-Р
Аноним 17/02/21 Срд 11:40:46 636054127
>>636047
В передаче по ютубу так астрофизик российский сказал.
Аноним 17/02/21 Срд 11:57:12 636057128
>>636054
Надеюсь эксперт астрофизик?
Аноним 17/02/21 Срд 13:06:44 636068129
Аноним 17/02/21 Срд 13:15:53 636069130
Аноним 17/02/21 Срд 16:50:55 636099131
Существуют ли миллисекундные Смегметары?
Аноним 17/02/21 Срд 17:27:06 636110132
Screenshot20210[...].jpg 249Кб, 720x1382
720x1382
Теперь слушай меня, червь бесхребетный. Следущий ТТВ будет Роберт Дункан эдишен, а следующий после него - Кристофер Томпсон эдишен. И уверяю тебя, твой говнофорс будет перемолот и уничтожен огромной массой правильно написанных терминов Магнетар. Сотни, тысячи магнетаров в одном ТТВ, и ещё больше их будет в другом. А третий ТТВ мы назовём в честь российского лингвиста и закрепим наш успех.
Теперь ПШЁЛ ВОН, лебезящий перед "авторитетами" бесхребетник!
Аноним 17/02/21 Срд 17:29:41 636111133
image.png 256Кб, 483x604
483x604
Аноним 17/02/21 Срд 17:58:48 636116134
>>636110
Кек, додик, ты не в ту сторону воюешь. Проснись. Речь идет о норме русского языка, а не английского.
Аноним 17/02/21 Срд 20:52:24 636138135
Какова вероятность существования миллисекундных Смегметаров? Ответьте мне.

Я запетнтовал название, если вдруг Смегметары реальны.
Аноним 17/02/21 Срд 20:52:48 636139136
Аноним 17/02/21 Срд 20:53:37 636140137
>>636110
>принимать английское написание за норму в русском
Ты и официально с двумя буквами пишешь?
Аноним 17/02/21 Срд 21:00:04 636145138
Аноним 17/02/21 Срд 21:01:31 636147139
>>636140
Магнетар - это изначально портманто, пришло из английского, специфический термин, с ХУЕВ ли там буква "и" должна появиться, уебище?
Аноним 17/02/21 Срд 21:04:49 636152140
>>636145
что руководство.
Аноним 17/02/21 Срд 21:05:49 636153141
d
Аноним 17/02/21 Срд 21:07:27 636154142
>>636041
И опять ни одного пруфа. Ни одного рассчета, измерения и математической выкладки или ссылки. "Яскозал" не от ананаса с борд, а от ученого, это все еще "Яскозал", если не приведены доказательства. Да, есть гипотеза, что вселенная замкнута. Как и гипотеза, что она плоская. Но без измерений это пустая болтовня, пусть хоть сам Альберт Эйнштейн восстанет и станет это говорить. А все проведенные измерения имеют недостаточно малую погрешность, в итоге диапазон N±n, Где N - значение измерения, а n - погрешность, включает как плоскую вселенную, так и замкнутую или гиперболоидную.
Аноним 17/02/21 Срд 21:47:45 636171143
Может ли вокруг солнца крутиться не девятая планета, а коричневый карлан чуть больше юпитера?
Аноним 17/02/21 Срд 21:49:58 636172144
>>636171
Вообще или в реальности? Вообще, конечно может. В реальности его наличие уже давно заподозрили бы, будь он близко - гравитационное влияние слишком велико. Ну или он на расстоянии тысяч астрономических единиц. Так что в принципе может.
Аноним 17/02/21 Срд 23:29:13 636194145
>>636172
>тысяч астрономических единиц
Не просто тысяч единиц, а ультрахолодный корич на тысячах единиц. Толку от такого нет абсолютно, жить там негде, т.к. обитаемая зона при такой низкой температуре (меньше 500К) не существует
Аноним 18/02/21 Чтв 02:23:08 636221146
Если создать вокруг Земли кольцо из частиц угля, то можно ли понизить температуру на экваторе? Типа запускаем несколько тысяч вагонов угля на экваториальную орбиту, они же должны создавать тень на экваторе, которая не будет пропускать свет?
Аноним 18/02/21 Чтв 06:24:57 636230147
Аноним 18/02/21 Чтв 06:27:37 636232148
>>636221
Только если дать кольцо как у Сатурна строго по эклиптике. Но вагонов понадобится не пара тысяч, а как бы не пара квадриллионов.
Аноним 18/02/21 Чтв 08:35:17 636240149
Аноним 18/02/21 Чтв 10:25:27 636254150
>>635980
Это типа петросян.жпг для зумерья, что ли? Нихуя не умею в эти ваши мемчики.
Аноним 18/02/21 Чтв 10:26:35 636255151
>>635980
Пиздец ебальник мерзкий, уноси это отсюда.
Аноним 18/02/21 Чтв 13:56:15 636281152
Аноним 18/02/21 Чтв 14:27:29 636289153
>>636281
Ты до сих пор копротивляешься, шизик?
Аноним 18/02/21 Чтв 15:31:00 636295154
Аноним 18/02/21 Чтв 17:18:17 636306155
Итак уже в 3 раз попытаюсь узнать КАКОЕ УСКОРЕНИЕ СВОБОДНОГО ПАДЕНИЯ В ДОТА 2??!?!??!??!
Аноним 18/02/21 Чтв 17:27:54 636307156
>>636306
Ускорение свободного падения в Дота 2 составляет 800.
Аноним 18/02/21 Чтв 18:06:54 636313157
Аноним 18/02/21 Чтв 18:27:07 636314158
>>636232
А как идея запускать реголит с Луны ? оттуда вагоны швырять легче
Аноним 18/02/21 Чтв 18:30:07 636315159
Аноним 18/02/21 Чтв 18:30:39 636316160
>>636314
Вместо неуправляемого говна и синдрома Кесслера задумайся о солеттах.
Аноним 18/02/21 Чтв 18:31:39 636317161
Аноним 18/02/21 Чтв 18:36:05 636318162
1158462496kacey[...].jpg 48Кб, 600x300
600x300
>>636316
Похуй на нижнюю орбиту, её надо спецом засрать чтобы не шпионили. Ты хочешь чтобы на орбите висел спутник, который с разрешением 1см на пиксель будет за всеми следить? Я вот не хочу чтобы с орбиты пялили на то как я ссу за гаражом, пусть лучше нижняя орбита вся будет в обломках, чем мою приватность еще и с орбиты нарушают. А потом еще суки 0.5 см на пиксель выкатят, а потом радарный спутник, который сможет с орбиты видеть есть ли у меня на машине все фары, зеркала, будут штрафовать за нарушение правил движения, когда я один на лесной дороге превышаю. Ты хочешь в этом мире жить? Чтобы с орбиты пиндосы могли число прыщей у тебя на ебале считать, когда ты на пляже загораешь? Лучше пусть никто там не будет нихуя иметь, но бонусом будет снижение температуры на всей земле.
Аноним 18/02/21 Чтв 18:45:23 636319163
>>636318
Ебать ты угловое разрешение.

>Лучше пусть никто там не будет нихуя иметь,
Ебать ты нехочуха.
Аноним 18/02/21 Чтв 18:51:16 636320164
>>636318
На это могу только сказать "нет".
Аноним 18/02/21 Чтв 18:51:50 636321165
>>636318
>Ты хочешь в этом мире жить?
Да, это охуенно же. Быдло прижмут, красота.
Аноним 18/02/21 Чтв 19:04:30 636322166
>>636321
Да тебя прижмут тоже.

>>636319
Старшип ебучий может запустить какой-нибудь оригами-джеймс вебб, который будет пырить вниз.
Аноним 18/02/21 Чтв 19:05:49 636324167
>>636322
>Да тебя прижмут тоже.
Меня не за что.
Аноним 18/02/21 Чтв 19:55:48 636326168
Аноним 18/02/21 Чтв 19:57:17 636327169
>>636326
ну типа кольцо сатурна бросает тень же, таким же макаром без сложного астроинженеринга запустить пыль и создать кольцо которое также будет понижать температуру на экваторе за счёт создания тени
Аноним 18/02/21 Чтв 20:04:00 636329170
image.png 145Кб, 250x250
250x250
>>636327
> будет понижать температуру на экваторе
Аноним 18/02/21 Чтв 20:18:40 636332171
image.png 296Кб, 880x660
880x660
>>636327
ровно на экватор не попадет
Аноним 18/02/21 Чтв 21:06:18 636341172
>>636329
Чел, там глобальное потепление во первых, во вторых Солнце светимость повышает и уже через 1 млрд лет температура на планете будет +60 по цельсию в среднем. Не только человек, но вообще все сложные биологические виды это не переживут. То есть разумная и сложная жизнь на планете этой сдохнет задолго до того как солнце войдёт в стадию красного гиганта и начнёт по очереди жрать планеты, ам меркурий, ам венера, ам земля. нееее, жизнь на земле начнёт подыхать гораздо раньше, все океаны выкипят уже через 1.2 млрд лет.
Аноним 18/02/21 Чтв 21:09:46 636344173
Какова вероятность, что вместо 9 планеты обнаружим Смегметар?
Аноним 18/02/21 Чтв 22:26:29 636376174
>>636341
За миллиард лет можно в L1 построить приличное такое поле солнечных батарей, перекрывающих десятки процентов прилетающего солнечного света. Причём намайненной энергии хватит на все возможные потребности вплоть до промышленного производства антиматерии. Так что если не хуёвничать - можно продлить земляшке жизнь и до стадии красного гиганта.
Аноним 18/02/21 Чтв 22:53:05 636402175
>>636376
Ага, щас. Тут лет через 50 планируется всемирный халифат с погружением в средневековье. Не будет никаких сфер дайсона и прочего.
Аноним 18/02/21 Чтв 23:29:15 636436176
Вопросачеры, а вы чувствуете себя сверхлюдьми, когда смотрите на Луну и примерно чувствуете представляете, где сейчас находится Солнце, а не "ыыы, смотри месяц на небе". Когда смотрите на Млечный Путь, вы осознаете, что смотрите в центр нашей галактики, а не "ыыы, смотри скока зведочек в одном месте". Когда сгорает метеор, вы думаете: "А ведь это всего лишь частица размером с песчинку, как будет выглядеть след от хуеты размером с кулак", а не "ыыы, звезда упала"?
Аноним 18/02/21 Чтв 23:35:55 636450177
image.png 15Кб, 800x560
800x560
Поясните! Допустим я хочу облететь марс и съебаться куда-нибудь подальше.
Но так получилось, что периапсис моей орбиты находится глубоко под поверхностью марса. И заметил я это уже очень близко от марса.
Есть шанс не разьебаться? Куда нужно давать вектор скорости или уже ничего не поможет?
Аноним 18/02/21 Чтв 23:36:14 636453178
>>636436
Нет. Но ощущение того, что ты ебаное ничего по сравнению с этим, присутствует.
Аноним 18/02/21 Чтв 23:37:57 636457179
>>636450
Если у тебя есть достаточно мощный движок и достаточно топлива (мощность и запас топлива зависит от расстояния, чем дальше, тем меньше требуется и то, и другое), то жечь надо перпендикулярно вектору твоей скорости, чтобы вектор тяги лежал в плоскости твоей траектории. Короче говоря, радиально.
Аноним 19/02/21 Птн 04:08:10 636617180
>>636457
Не будет ли ретроберн более эффективным?
Аноним 19/02/21 Птн 04:38:48 636618181
Посоны, такик вопросы, нигде толком ничего не нашел
1 Если с газового гиганта сдуть атмосферу, то что будет с ядром? Оно станет типа каменистой планеты но с неебовейшей гравитацией?
2 Постоянно слышу что Нептун и Уран называют ледяными гигантами. Как это правильно понимать?
3 В теории любая планета при наличии невероятных условий может стать звездой?
Аноним 19/02/21 Птн 04:40:17 636619182
>>636617
Эх ты, как ты в КСП играешь, а ещё не понял такую мелочь важную?
Будешь жечь реверс ретробёрн, твоя траектория уползёт ещё глубже в глубины МарсаДюны и единственное торможение, которое тебя ждёт, литосферное.
Аноним 19/02/21 Птн 09:18:40 636640183
>>636617
Скачай КСП, для начала. Потом найди уравнение движения материальной точки в поле тяготения и разберись что такое векторы.
Аноним 19/02/21 Птн 09:24:48 636641184
>>636618
1. Может быть. В принципе если начать сдувать атмосферу, то сначала будет газифицироваться лежащий глубже жидкий водород, а вот что будет с таким объемом металлического водорода после того, как снимешь давление, никто не знает. Может быть и бабах.
2. Они состоят не как Юпитер и Сатурн в основном из водорода и гелия, но еще и с немалой примесью аммиака, воды, метана и других подобных бинарных соединений элементов начала таблицы. И в глубине у них скорее всего не металлический водород, как у газовых гигантов, а экзотические фазы льда, вроде того же Льда-18.
3. Ну да, любая. Если массы наберет достаточно. Нет, Юпитеру очень далеко до звезды. Разница между Юпитером и самым дохлым красным карликом примерно такая же, как между Землей и Юпитером. Более реалистично если так можно сказать будет выглядеть вариант с превращением полосатого в недозвезду - коричневый карлик, в котором настоящего протон-протонного звездного синтеза не идет, но вяло тлеет дейтерий и тритий, разогревая карлана до температуры в 300-600К.
Аноним 19/02/21 Птн 11:18:52 636661185
>>636641
Спасибо анон за развернутый ответ.
Хотелось насчет второго только уточнить.
Вот дефолтный газовый гигант легко представить - очень грубо говоря, снаружи газ, который плавно переходит в жидкость. А как правильно описать ледяной гигант?
Аноним 19/02/21 Птн 11:31:16 636662186
>>636661
We need to go deeper
Газовый гигант:
1. Газообразный водород, чутка гелия
2. Газообразный водород погорячее, много NH3, H2O, CH4, NH4HS и тд
3. Сжиженный водород
4. Металлический водород
5. Железо-каменное ядро
Ледяной гигант:
1. Газообразный водород, но гораздо больше NH3, H2O, CH4, NH4HS и прочего такого
2. Перечисленные пунктом выше вещества в жидкой фазе, скорее всего водный океан с растворенными в нем веществами
3. Ледяная оболочка, лед, разумеется, не простой, а его фазовые модификации
4. Железо-каменное ядро
У ледяных гигантов в силу их меньшей массы и нахождения на окраинах солнечной системы меньше водорода в составе, потому что все забрали газовые гиганты и не хватает гравитации на удержание мелких и быстрых молекул водорода, поэтому гораздо больше его соединений с другими элементами.
Аноним 19/02/21 Птн 11:59:31 636670187
>>636662
Не смущай анона, у тебя ошибки.
Газовый гигант - модель с 4-мя слоями (самая свежая)
1- наружный слой, газообразный, водород-гелий, доля NH3 и H2S от 1/100 000 до 1/10 000
2- второй слой, жидководородный (гелий - газ или в каплях тоже) , присутствует ионная проводимость, примесей NH3 и H2S, вероятно, больше
3 - металлический водород (гелий смешан), тяжёлых элементов уже сильно больше чем в слое 1 и 2, отлично проводит ток
4 - ядро, много тяжёлых, мало водорода-гелия. ядро не из камня, "тяжёлые элементы" здесь это на 80% кислород и азот (в виде соединений)
Короче, любой газовый гигант суть планета-океан (водородный)

Ледяной гигант:
1- наружный слой, газообразный, водород-гелий, доля H2S от 1/100 000 до 1/10 000 (но несколько выше, чем для газовых гигантов) , доля NH3 < 1/100 000 (ниже чем для газовых)
2- второй слой, жидководородный (гелий - газ или в каплях тоже) , присутствует ионная проводимость, примесей NH3 и H2S, вероятно, больше, H2S всё ещё сильно больше чем NH3. На Уране слоя 2 нет, на Нептуне есть, но тонкий.
3 - океан воды-метана-аммиака. Мало серы, много азота, почти весь азот (в виде соединений) растворён в океане, поэтому слои 1 и 2 обеднены азотом очень сильно.
4 - ядро из камня

Аноним 19/02/21 Птн 12:01:40 636671188
>>636670
быстрофикс, доля H2S от 1/100 000 до 1/10 000, а доля NH3 от 1/10 000 до 1/1000 для газовых. Для ледяных так же но только для H2S
Аноним 19/02/21 Птн 12:03:46 636672189
>>636670
>"тяжёлые элементы" здесь это на 80% кислород и азот (в виде соединений)
и углерод
фикс
Аноним 19/02/21 Птн 12:26:02 636676190
>>636618
1) сдуть всю атмосферу - будет каменистая планета. Или планета-океан (если ты, сдув водород и гелий, охладишь планету чтобы сконденсировался водяной пар).
Сдуть водород, но не гелий, в принципе, тоже можно. Гелий тяжелее, он будет сдуваться (если это идёт через нагрев молекул) вторым. Но сдувающийся водород будет увлекать часть гелия, т. к. молекулы (атомы) этих двоих не сильно отличаются (оба лёгкие). Это процесс гидродинамической потери атмосферы. А т.к. водорода в 5 раз больше чем гелия, то он почти весь его и захватит с собой, унесёт.
Тем не менее, в редких случаях можно оставить часть гелия, из большого-пребольшого газового гиганта (я бы сказал, Юпитер маловат, нужно 2-5 масс Юпитера) получится большой ледяной, с очень тяжёлым ядром в 40-100 масс Земли и гелиевой атмосферой, сильно загрязнённой H2S. Если ты возьмёшь Сатурн, то гелия так мало будет, что получится не гигант вовсе, а супер-планета-океан, массой в 15-20 земных и тонкой (меньше 500 атм.) гелиевой атмосферой. Юпитер - что-то между.
2) ледяной гигант = состоит изо льда. Процентов 80-85 массы приходится на C, O и N, а на H-He всего десяток процентов. Никакой там не лёд, конечно, а что-то такое полужидкое (как мягкий воск) или жидкое, горячее (4000 градусов) и плотное вещество, в нём O, C, N и несколько процентов H. И из него составлена мантия, а сверху атмосфера, а снизу ядро с железом, кремнием. В мантии 60-70% массы планеты.
3) да, любая, массы добавь и начнутся реакции. Минимальный порог для термоядерного синтеза (=звезда) это всего 10-12 масс Юпитера. Только такая звезда (дейтериевая) пропыхтит десятки миллионов лет всего (дейтерия очень мало) и сдохнет, и потом миллиарды лет по-инерции будет остывать (как белый карлик по сути). Это коричневый карлик. При массе в 70 юпитеров, звезда вслед за дейтерием переработает и литий-7 (но его ещё меньше чем дейтерия), а при массе в 80 юпитеров загорится водород и будет гореть долго, как у нормальных звёзд.
Аноним 19/02/21 Птн 12:45:44 636677191
Раз уж вы про состав звезд-неудачников заговорили, давайте тогда еще поговорим про Смегметары.

Как думаете, из чего состоит Смегметар?
Аноним 19/02/21 Птн 13:11:42 636682192
Аноним 19/02/21 Птн 13:51:59 636687193
>>636676
>>636670
Жесть аноны, спасибо за ответы, теперь есть о чем поразмышлять.
Даже не думал что так развернуто ответят, думал как обычно мудаком назовут и нахуй пошлют.
Аноним 19/02/21 Птн 13:53:46 636688194
>>636676
> будет каменистая планета
Ну ведь типа Земли, Венеры все равно из этого не выйдет? Я так понимаю будет очень плотный шарик с лютой гравитацией.
Аноним 19/02/21 Птн 14:43:37 636699195
>>636688
Ну, сложно сказать. Давление упадёт, начнётся расширение всех этих...горных пород. Но! В ядре гиганта температура высока. Если ты быстро сдуешь, миллионов за 10 лет, давление упадёт, а температура почти нет, не успеет. В итоге вся планета будет горячая (10-15 тыс. градусов внутри) и значит жидкая. Собственнно, при таком раскладе никаких механических напряжений внутри не возникнет, плавно остынет, возникнет кора и т. д.
Но есть такой гипотетический сценарий - "Хтоническая планета". Там время больше и, быть может, сначала часть планеты застынет, потом сдуется атмосфера. Короче, если так, то будут землятресения, от постепенного высвобождения механических напряжений в коре. Но это, на мой взгляд, маловероятно, т.к. хтоническая планета получается из очень горячего гиганта (звезда раздуватся до красного гиганта и прожаривает свою планету), какое уж тут "холодное ядро"?
https://ru.m.wikipedia.org/wiki/%D0%A5%D1%82%D0%BE%D0%BD%D0%B8%D1%87%D0%B5%D1%81%D0%BA%D0%B0%D1%8F_%D0%BF%D0%BB%D0%B0%D0%BD%D0%B5%D1%82%D0%B0
Аноним 19/02/21 Птн 14:44:28 636700196
Аноним 19/02/21 Птн 16:27:32 636712197
Аноним 19/02/21 Птн 16:36:07 636713198
Посоны, всем известно что гравитация зависит от массы, ну тип чем тяжелее, тем сильнее гравитация.
Может ли гипотетическая суперземля иметь сходную гравитацию с нашей Землей, или даже меньшую? Что-то вроде объем больше, но но плотность меньше чем на земле? Ну я думаю поняли что я имею ввиду, прошу прощения за сумбурность.
Аноним 19/02/21 Птн 16:42:54 636715199
>>636713
суперземля не может иметь меньшую, так как для этого нужно иметь намного меньшую плотность, это уже тогда не будет земля
Аноним 19/02/21 Птн 16:52:28 636717200
>>636715
Ну я хз, может состоит из более легких пород и прочее.
Аноним 19/02/21 Птн 16:52:46 636718201
>>636713
В некоторых пределах - да. Но она не будет сильно больше по размерам. Раза в полтора по массе, разве что.
1) безъядерная планета, железо в составе соединений(силикатов) в мантии. Ядра нет, но плотность мантии чуть выше, общая плотность чуть чуть ниже, проценты.
2) углеродная планета. не такая суперуглеродная, как в говностатье на википедии, а средней "углеродности". в итоге это такая же землеподобная планета, но в мантии полно алмазов, алмазы не сжимаются и сильно разуплотняют глубинные породы. Общая плотность, ну, может, процентов на 10 ниже
3) углеродная планета с низким содержанием железа. сейм + соглы (1+2), процентов на 12-15 ниже плотность.

Вот и выходит, что в диаметре отличия будут, но небольшие
А если взять планету-океан с 50% воды (сорт оф Гига Чад Ганимед) то там уже другой расклад, общая плотность 2-2,5 вместо 5-6, масса 4 земных и диаметр в 2 раза больше. Но в чём прикол планеты-океана? Это хуета.
Аноним 19/02/21 Птн 17:34:59 636723202
>>636718
> планета океан
По моему прикольная тема. И все равно же под океаном есть рельеф, интересно узнать как породы под воздействием жидкости меняются.
А вообще анон ты гений, одно удовольствие тебя читать.
Аноним 19/02/21 Птн 18:59:32 636737203
Аноним 19/02/21 Птн 19:01:36 636738204
Почему радиотелескопы не находят никаких следов внеземной цивилизации?
Аноним 19/02/21 Птн 19:02:24 636739205
Аноним 19/02/21 Птн 19:05:36 636740206
Аноним 19/02/21 Птн 19:10:51 636741207
Аноним 19/02/21 Птн 19:11:50 636742208
>>636740
Не успели появиться.
Вселенная очень молодая. Только-только появилось 3-е поколение звезд. До 3-го было невозможно появление органическое жизни.
Аноним 19/02/21 Птн 19:23:43 636744209
>>636742
> Только-только
Речь идёт о миллиарде лет, да?
Аноним 19/02/21 Птн 19:25:56 636745210
>>636741
Очередной пук нейтронной звезды или что-то в таком духе
Аноним 19/02/21 Птн 19:26:55 636746211
>>636744
Это ничто в плане эволюции вселенной и появления живых организмов.
Аноним 19/02/21 Птн 21:14:30 636769212
232323232.png 34Кб, 686x216
686x216
А рф вообще приглашал кто то ? Или как обычно " вы нас не пригласили, но мы сделаем вид что сами не хотим"
Аноним 19/02/21 Птн 21:15:52 636770213
>>636769
Предлагали сделать какие-то мелкие детали, но газоген развыебывался и обиделся, что не дали делать полноценный модуль
Аноним 19/02/21 Птн 21:23:31 636780214
>>636770
>не дали делать полноценный модуль
лол а покммок осилил бы? Науку никак не склепают, федерация\орел показали тогда же, когда и драгон и до сих пор дальше прототипа не ушли.
Аноним 19/02/21 Птн 21:25:08 636782215
>>636770
Обиделся, потому что за свои бабки надо было делать. Они же хотели на халяву.
Аноним 19/02/21 Птн 21:26:23 636784216
>>636782
>что за свои бабки надо было делать
Тогда совсем не понятно, рф в ИТЕР учавствует и вполне себе за свои бабки, никто не бухтит на этот счет а тут тоже могли бы за свои
Аноним 19/02/21 Птн 21:34:08 636790217
>>636784
Там копейки. А здесь 200-300 лимонов это накладно для роскосого. А если бы наса насыпало денег и дало добро на постройку пару модулей, как гейропцам, а не какой-то затюханый шлюз.
Аноним 19/02/21 Птн 21:38:09 636795218
>>636790
Можно было согласится построить шлюз и насрать туда где нить в скрытое место
Аноним 19/02/21 Птн 21:46:04 636801219
>>636790
Насколько реально - "Дмитрий Рогозин сказал, что Россия может построить окололунную станцию без NASA"
Аноним 19/02/21 Птн 21:53:52 636805220
>>636801
В лучшем случае кастрированный Мир на НОО, без постоянного присутствия на станции.
Аноним 19/02/21 Птн 22:05:37 636810221
Гравитационного маневра вопрос.
1. Какого прироста скорости можно добиться, если выполнить подобный маневр около черной дыры или нейтронной звезды?
2. Насколько сложно выполнить такой маневр и какие подводные?
Аноним 19/02/21 Птн 22:07:21 636812222
Академики обьясните.
Есть ли сегодня теории о том, что образует массу в черной дыре? Ведь сингулярность, скорее математический термин, а не физический.
Аноним 19/02/21 Птн 22:11:34 636814223
>>636812
суть в том что при такой плотности на состав массы поебать
но если посмотреть на состав объектов из которых получаются дыры - там в основном водород
Аноним 19/02/21 Птн 22:12:46 636815224
>>636801
Обещать может любую хуйню, это не мешает ему пилить бабосик и покупать яхты
Аноним 19/02/21 Птн 22:15:23 636816225
>>636814

Но нейтронная звезда состоит из нейтронов, а не водорода.
Так как масса не исчезает, следовательно мне любопытно, о материи ли вообще идет речь, когда доходит дело до таких чудовищных плотностей.
Аноним 19/02/21 Птн 22:24:05 636817226
>>636810

У ЧД маневр выполнить очень сложно. Как пробежать по острию бритвы и не порезаться при этом.
Если у дыры акреционный диск - ты сгоришь.
Сам маневр может тебе стоить сотни тысяч лет во внешней вселенной.
Крохотная ошибка в расчетах - ты сорвешься за горизонт событий.
Охуенно опасная процедура.

С нейтронной звездой не лучше. Чудовищное магнитное поле породит чудовищное электрическое.
Опять же - сгоришь. Плюс - дикая радиация. Не лучший выбор для гравитационного маневра.

Самый возможный вариант - белый карлик, желательно остывший.

Приращение в скорости - потенциально десятки тысяч километров секунду у ЧД. У белого карлика - сотни километров в секунду.
Аноним 19/02/21 Птн 22:27:16 636818227
а что если подрочить в черную дыру?
Аноним 19/02/21 Птн 22:32:35 636820228
>>636817
Ну вряд ли всякие мелкие кораблики будут делать манёврики у ЧД. Разве что огромный корабль поколений с телами или виртуальными образами людей и цифровым днк. Он будет дохуя забронированным от радиации и иметь ИИ для манёвра. Такой корабль можно послать в спутниковую галактику млечного пути, в шаровое скопление какое-нибудь.
Аноним 19/02/21 Птн 22:51:37 636828229
>>636817

У нейтронной звезды - без шансов выполнить маневр.
Эта сука маленькая всего 12-15 км, у нее в принципе нет безопасных мест.
Аноним 19/02/21 Птн 22:56:48 636830230
>>636820

Энергии слишком большие. Не получится защититься.
Я не уверен, что даже ИИ под силу провести корабль в нескольких десятках метров от горизонта событий вращающейся ЧД. Мы слишком мало знаем об этом.
Вполне возможно, что это не некая идеальная линия, за которую не стоит залетать, а хаотично изменяющаяся область движение которой невозможно предсказать.
Аноним 19/02/21 Птн 22:57:24 636831231
Поиграв три года в огурцы и наебавшись со взлетом с Евы и Теллумо из GPP задался вопросом - а возможно ли вообще взлететь с Венеры? Учитывая, что на Еве 5 атмосфер, а на Венере все 90. Движки работать будут вообще?
Аноним 19/02/21 Птн 23:01:13 636833232
>>636831

При нынешних технологиях нет, невозможно.
Аноним 19/02/21 Птн 23:02:15 636834233
>>636833
А что мешает, если не секрет? Движки не заработают или там что еще?
Аноним 19/02/21 Птн 23:04:14 636835234
>>636831
Взлететь можно, движки работать будут (если в камере сгорания давление под хотя бы 200 атмосфер), но импульс будет очень хуёвый и сопротивление атмосферы астрономическое.

Но зачем это вообще делать? Надуваешь аэростат и легко поднимаешься на 50 км вверх. В огурцах тоже первая ступень на пропеллерах очень хорошо работает.
Аноним 19/02/21 Птн 23:06:36 636836235
>>636834

Гравитация такая же, как на Земле, очень плотная атмосфера, очень высокая температура.
То есть, только многоступенчатая ракета, а значит - полноценный космодром.
Но самое главное, и ты об этом сказал - атмосферное давление. Давление углекислого газа у поверности такое, что он считай уже и не газ, а скорее жидкость. Это как на Земле запускать ракету с глубины в 1 км.
Таких технологий просто нет.
Аноним 19/02/21 Птн 23:33:22 636846236
>>636836
Как нет? Привязать шарик к ракете, она поднимется и там можно движки запустить будет.
Аноним 19/02/21 Птн 23:34:32 636847237
>>636846
Странно что Илон Маск так не догадался еще сделать.
Аноним 19/02/21 Птн 23:46:33 636850238
>>636847
Не Илон Маск разрабатывает хуйню, а его инженегры
Аноним 19/02/21 Птн 23:48:39 636851239
image.png 47Кб, 610x383
610x383
>>636850
но он сам инженер
Аноним 20/02/21 Суб 05:15:13 636882240
>>635533 (OP)
>Можно упасть в пузырь Ай-Тойона, NASA уже почти надула его.

Ребять, я тут мимо-прохожу, но почему Ай-Тойон? Уоскуй братка
Аноним 20/02/21 Суб 08:10:31 636898241
1921000.jpg 76Кб, 828x816
828x816
Господа, посоветуйте хороших по вашему мнению фильмов про космос или в антураже космоса.
Их смотрел я много, но их мне всё мало, хочу ещё.
Желательно нашего времени производства, то есть до фильмов до 1961 и/или чёрно-белых, пожалуй, не нужно, а известные вам новинки - наоборот советуйте.
Аноним 20/02/21 Суб 08:40:44 636900242
Аноним 20/02/21 Суб 08:45:15 636901243
>>636898
Космическая одиссея
Луна (с Сэмом Рокуэллом)
Интерстеллар
Аноним 20/02/21 Суб 08:52:50 636904244
>>636900
>Planetes
Уныло. Не зашло вообще. 2 серии через силу посмотрел и дропнул.
>>636901
Всё это видел.
Аноним 20/02/21 Суб 09:29:42 636907245
image.png 209Кб, 749x485
749x485
image.png 563Кб, 744x483
744x483
Что за хуйня с газогеном?
Аноним 20/02/21 Суб 09:59:07 636919246
>>636904
Ну и пшел нахуй отседава тогда.
Аноним 20/02/21 Суб 10:08:23 636922247
>>636904
Ну если планетес - уныло, то тогда иди смотри Звездные Войны или Трансформеры. Там много пиу-пиу, другого экшена - одним словом для таких как ты весело.
Аноним 20/02/21 Суб 10:11:02 636923248
>>636922
Анимешник, успокойся
Аноним 20/02/21 Суб 10:14:06 636925249
Аноним 20/02/21 Суб 10:23:24 636927250
>>636812
Массу черной дыры образует все что в нее когда-либо попало. Другое дело, что в центре черной дыры вещество достигает такой плотности, а пространство-время - такой кривизны, что абсолютно все наши теории, включая в первую очередь ОТО, там ломаются и перестают работать в принципе. Поэтому и говорят о сингулярности - об особенности. Центр черной дыры - особенное место, в котором все известные нам законы физики неприменимы. Для описания недр черной дыры нужна квантовая теория гравитации, а ее пока нет и вообще непонятно, когда ее построят (и построят ли).
Аноним 20/02/21 Суб 10:57:04 636946251
>>636907
Шутник, картиночки рассчитаные на аудиторию " унизил пиндосов, дал пососать"
Аноним 20/02/21 Суб 12:20:57 636966252
>>636700
a 4to togda prupfovie oxfordovskovo slovaria sovremenogo englisha?
Аноним 20/02/21 Суб 12:23:05 636968253
Аноним 20/02/21 Суб 16:26:42 637009254
boocover.jpg 22Кб, 200x305
200x305
>>636882
Тому што "Ай-Тойон" звучит почти как "Альбукерке", а "Продавец воздуха" - точь в точь "Илон Маск".
на пике Ноздря Ай-Тойона
Аноним 20/02/21 Суб 16:27:34 637010255
>>636966
MAGNEeeeeTAR slovo rysskogo yazika, manya.
Аноним 20/02/21 Суб 17:08:07 637027256
А можно, включив движки, зависнуть около горизонта событий черной дыры, что бы улететь в будущее на миллиарды лет?
Аноним 20/02/21 Суб 17:08:56 637028257
Аноним 20/02/21 Суб 17:11:20 637030258
>>637027

В теории можно.
На практике, все упирается в массу ЧД. Чем выше масса, тем проще это делать.
Кажется у Торна "В складках времени", для ЧД массой в миллион солнечных, требовались движки - способные создать ускорение в десятки миллионов Ж.
Аноним 20/02/21 Суб 17:18:57 637035259
Если до нас долетят пришельцы и захотят нас завоевать - будут ли у землян хоть какие то шансы?
Аноним 20/02/21 Суб 17:23:04 637038260
>>637035
Начнем с того, зачем им лететь? Ресурсы? Цивилизации, освоившей межзвездные перелеты, не хватает золота/осмия/воды/чего то еще? Вряд ли. Рабы? Опять-таки, зачем. Просто незачем завоевывать, слишком много ресурсов уйдет на завоевание, а оно ведь еще и ядерным оружием кусаться будет.
Аноним 20/02/21 Суб 17:26:10 637039261
>>637038

Может у них хобби такое.
Аноним 20/02/21 Суб 17:29:26 637040262
>>637039
Ну тогда неизвестно, насколько все может быть с их развитием. А то будет как в том рассказе, когда прилетели какие-то инопланетяне, которые освоили варп-драйв раньше электричества, и вообще межзвездные перелеты открывали очень рано, только люди вкачали другие ветки, попытались завоевать Землю с воздушными шарами, горшками с греческим огнем и мушкетами, а получили в ответочку сверхзвуковые истребители, управляемые ракеты и автоматическое огнестрельное оружие.
Аноним 20/02/21 Суб 17:38:01 637044263
>>637040

У них все прекрасно с развитием, поскольку они подчинили себе энергию качественного иного порядка.
Энергия - все. Коммунизм.Терраформирование планет. Управление климатом.
Что угодно станет доступно.
Если (ЕСЛИ) цивилизация, способная к перемещению между звездами - вдруг решит нас завоевать, у нас не будет НИ ЕДИНОГО шанса. Как нет их у аборигенов против авианесущей группировки.
Аноним 20/02/21 Суб 17:40:05 637045264
>>637038
>Рабы? Опять-таки, зачем.
Что значит зачем? Человеческий мозг - это очень совершенный компьютер. Вполне можно использовать в качестве рабов, если оптимизировать управление вычислительной мощностью моска.
>а оно ведь еще и ядерным оружием кусаться будет.
Эти ядерные пуки неочем. Вон сраный небольшой челябинский метеорит взорвался с силой как 3 бомбы в Хиросиме.
Аноним 20/02/21 Суб 17:40:54 637046265
>>637044
> коммунизм
> какое-то развитие
Придётся тебе выбрать одно.
Кстати, нахуя ты отвечаешь на свой вопрос?
Аноним 20/02/21 Суб 17:42:17 637047266
>>637044
Ты мыслишь человеческими категориями. Может им вообще неизвестна концепция войны. Или для них война это соревнование в лепке глиняных плюмбусов на скорость. Или у них принято воевать на заранее оговоренной территории до первой крови. Или наоборот экстерминатус вольфрамовой стрелой, разогнанной до 0,999с. Поэтому и ответить на первоначальный вопрос абсолютно невозможно.
Аноним 20/02/21 Суб 17:43:08 637048267
>>637046
Читай больше умных книжек, не будь быдлом.
Аноним 20/02/21 Суб 17:46:12 637049268
1613832370611.jpg 992Кб, 991x1494
991x1494
>>637048
А, так ты ретранслируешь высеры чьего-то моска прямо в итт тред? Да и ещё и считаешь себя небыдлом?
Аноним 20/02/21 Суб 18:05:28 637057269
>>637047

Вопрос был вполне конкретный. Было допущение "Если захотят завоевать".
Если захотят - завоюют. Будет какая-нить партизанская хуйня.
Но, с другой стороны, я уверен, что подчинить себе энергию которая сделает доступной перелеты между звездами, совершенно невозможно для качественного иного социального и политического уклада.
Общество потребления до звезд не долетит. Аватар хуета полная, цивилизации, способной за 6 лет добраться до Альфы Центавара - нахуй не упал онаптаниум.
Аноним 20/02/21 Суб 18:07:48 637059270
>>637057
Цивилизация именно поэтому и добралась до альфы центавра, что ей нужен был анобтаниум. Хотя возникает вопрос, почему после добычи первых образцов не начали его синтезировать самостоятельно, это да.
Аноним 20/02/21 Суб 18:10:55 637061271
>>637059

Там движки с топливом из антиматерии. О чем тут говорить.
Сколько щаз стоит синтезировать 1 грамм антиводорода - 2 или 20 триллиона баксов. Не помню уже, но - да хуя.+
Не, Кэмерон очень снисходителен к человечеству.
Аноним 20/02/21 Суб 18:31:56 637068272
>>637061
Двести лет назад алюминий стоил дороже золота. Будет спрос - будет производство в больших объемах, а значит и удешевление.
Аноним 20/02/21 Суб 18:36:32 637073273
>>637061
>Сколько щаз стоит синтезировать 1 грамм антиводорода - 2 или 20 триллиона баксов. Не помню уже, но - да хуя.+
Если измерять в киловатт-часах средней российской АЭС, то чуть меньше миллиона долларов.
А вот почему над повышением КПД выработки хотя бы до 20% до сих пор не трудятся все топовые мировые НИИ - это уже вопрос к приоритетам.
Аноним 20/02/21 Суб 18:38:14 637074274
>>637073
А смысл? Ты его даже хранить не сможешь никак.
Аноним 20/02/21 Суб 18:38:36 637075275
Почему вселенная расширяется?
Аноним 20/02/21 Суб 18:43:06 637076276
>>637073

Пруф. Насколько я знаю, щаз цена 1 грамма антиводорода около 10 триллиона баксов и связано это именно с энергозатратами.
Аноним 20/02/21 Суб 18:46:10 637078277
>>637075

Потому что есть темная энергия, которая отвечает за это расширение.
Аноним 20/02/21 Суб 18:47:25 637080278
>>637078
каким образом? что она делает?
Аноним 20/02/21 Суб 19:03:24 637089279
>>637080
Если бы я знал, давно получил бы Нобелевку.
Аноним 20/02/21 Суб 19:18:17 637093280
>>637074
>А смысл? Ты его даже хранить не сможешь никак.
https://nplus1.ru/blog/2018/03/02/antimatter-truck
>уже в 2011 году им удалось продержать атомы антиводорода в течение 1000 секунд (чуть меньше 17 минут), а отдельные антипротоны сейчас могут храниться более одного года.

>>637076
>Пруф.
Посчитай, сколько джоулей в грамме антиводорода, и переведи в квтч по курсу 1руб/квтч. Столько стоил бы этот грамм, если бы кпд перегонки электричества в антиматерию был 100%.
Даже в случае кпд 5% это всё ещё было бы рентабельно. Но пока что этот кпд - стотысячные доли процента, потому что производятся эти антипротоны на исследовательских установках, которые для промышленной генерации антивещества не создавались, а процесс этой генерации никто толком не оптимизировал, опытные установки уровня ИТЭР не строил.
Аноним 20/02/21 Суб 19:49:49 637099281
>>637073
>А вот почему над повышением КПД выработки хотя бы до 20% до сих пор не трудятся все топовые мировые НИИ
Ровно по той же причине, по которой в середине 18-го века никто не занимался разработкой технологии производства 98-го бензина.
Аноним 20/02/21 Суб 20:00:35 637104282
>>636904
> Всё это видел.
Тогда ты видел все.
Аноним 20/02/21 Суб 20:03:29 637105283
>>637040
Ссылку на рассказ в студию, позязя
Аноним 20/02/21 Суб 20:11:51 637108284
>>637047
> Ты мыслишь человеческими категориями. Может им вообще неизвестна концепция войны. Или для них война это соревнование в лепке глиняных плюмбусов на скорость. Или у них принято воевать на заранее оговоренной территории до первой крови. Или наоборот экстерминатус вольфрамовой стрелой, разогнанной до 0,999с. Поэтому и ответить на первоначальный вопрос абсолютно невозможно.
Хуйню пишешь. Без конкуренции невозможно развитие, а цивилизация развившаяся до уровня космических перелетов должна быть охуеть какой воинственной, иначе зачем эволюционировать из одноклеточного в сорт оф человека.
Аноним 20/02/21 Суб 20:23:51 637112285
>>637105
Гарри Тердлав "Тупик".
Аноним 20/02/21 Суб 20:25:57 637113286
>>637108
>Без конкуренции невозможно развитие
В условиях Земли. Как там на других планетах мы понятия не имеем.
Аноним 20/02/21 Суб 21:11:56 637126287
>>637113
> В условиях Земли. Как там на других планетах мы понятия не имеем.
Ага, а еще там физика и химия другие.
Аноним 20/02/21 Суб 21:15:03 637128288
>>637113

Закон сохранения энергии - фундаментальный закон вселенной. А значит, работает на любой планете
Для биологических систем - он будет работать при любых раскладах.
Жри других - что бы выжить. Только это - залог эволюции биологической системы.
Цивилизация, возможно, может прийти к пацифизму на определенном этапе развития. Но за плечами любой цивилизации, будут миллионы лет эволюция, пожирания друг друга и борьбы за жизненное пространство.
Аноним 20/02/21 Суб 21:42:37 637136289
>>637093

Некорректное сравнение.
В 1 грамме антиводорода столько же энергии как в 1000 тонн угля. Но уголь лежит в земле, а антиводород - нет. И дело не в КПД. Дело в затраченной энергии на синтез. И там, какая то астрономическая цифра. 20 000 триллионов КВт/ч, если мне не изменяет память.
Безусловно, я допускаю мысль, что технология будет совершенствоваться. Но до промышленного производства антиматерии нас отделяют столетия технологического. Я в этом абсолютно убежден.
Какие при этом нас ждут социальные потрясения, как это изменит мировую экономику и политические системы - не могу даже предположить.
Но для того, что бы полететь к звездам, человечеству придется измениться.
Это - неизбежно.
Аноним 20/02/21 Суб 23:27:52 637161290
image 1Кб, 200x119
200x119
Аноним 20/02/21 Суб 23:45:38 637164291
Какие проблемы стоят перед человечеством, которые не позволяют нам начать добычу полезных ископаемых с астероидов? Кроме экономических.
Аноним 20/02/21 Суб 23:48:59 637165292
>>637164
Отсутствие технологий по разработке астероидов в принципе.
Аноним 20/02/21 Суб 23:55:47 637168293
>>637165
А чем астероид отличается от простого камня?
Аноним 20/02/21 Суб 23:57:45 637169294
>>637168
А проживание в глубоком космосе? Или может быть мы умеем буксировать куски породы массой в 1012 - 1015 тонн? К тому же бессмысленно спускать все это на Землю, а надо перерабатывать в космосе, этого тоже делать не умеем.
Аноним 21/02/21 Вск 00:14:57 637171295
>>637169
Ну я же не прошу, сразу на Психею разрабатывать, мелкие околезмные астероиды ну пускай там масса будет 50-100 тысяч тонн перевести на орбиту и начать разрабатывать.
Какие вот с такими астероидами проблемы?
Аноним 21/02/21 Вск 00:20:35 637178296
>>637168
Простой камень на земле лежит, а астероид в космосе, в экстремальной среде. он крутится, нагревается охлаждается, двигается с огромной скоростью вокруг солнца по сложной орбите, за него не всегда можно уцепиться. Часто астероид это не камень, а глыба камней или песка, сцепленная гравитацией. Представь себе глыбу сахара размером с гараж, которая очень условно скреплена - ты в неё крюк втыкаешь, он тонет или же насквозь проходит и выходит не зацепляясь.

>>637171
>перевести на орбиту
Там нужно дохуя топлива чтобы изменить орбиту большого астероида, может статься что доставляя это топливо в космос с земли, чтобы загрузить в чудесный изменитель орбит астероидов, ты потратишь больше денег, чем в итоге заработаешь разрабатывая астероид на орбите.
Аноним 21/02/21 Вск 01:29:04 637194297
image 58Кб, 640x362
640x362
>>637178
>Простой камень на земле лежит, а астероид в космосе, в экстремальной среде. он крутится
Ну стыковка с вращающимся объектом уже не новость.

>нагревается охлаждается
А что не так с нагревом и охлаждением?

>двигается с огромной скоростью вокруг солнца по сложной орбите
Скорость относительна ,но да конечно 15-20км/с это проблема.

>за него не всегда можно уцепиться. Часто астероид это не камень, а глыба камней или песка, сцепленная гравитацией. Представь себе глыбу сахара размером с гараж, которая очень условно скреплена - ты в неё крюк втыкаешь, он тонет или же насквозь проходит и выходит не зацепляясь.
Ну да, это тоже проблема. Можно что-то похоже на пик1 соорудить.Или искать астероиды, которые не будут сыпаться как сахар.

>Там нужно дохуя топлива чтобы изменить орбиту большого астероида, может статься что доставляя это топливо в космос с земли, чтобы загрузить в чудесный изменитель орбит астероидов, ты потратишь больше денег, чем в итоге заработаешь разрабатывая астероид на орбите.
Да. Может но мы же умные нет (люди) рассчитаем будет ли выгодно или нет.

Я вот читаю на эту тему, вот какие-то бактерии которые добывают полезные ископаемые, их уже тестируют в невесомости.
Ссылка:https://www.popmech.ru/science/news-642383-bakterii-mogut-dobyvat-poleznye-iskopaemye-dazhe-iz-asteroidov/

Или на Психею NASA утвердила полёт.
Мне даже кажется, что на крупные Астероиды будет легче, ибо о них гораздо больше известных данных будет, но с другой стороны, тогда придется добывать прямо там, а это диаметрально противоположный курс, такой астероид ты не отбуксируешь.
Ссылка на миссию:
https://ru.wikipedia.org/wiki/Psyche_(%D0%BA%D0%BE%D1%81%D0%BC%D0%B8%D1%87%D0%B5%D1%81%D0%BA%D0%B8%D0%B9_%D0%B0%D0%BF%D0%BF%D0%B0%D1%80%D0%B0%D1%82)#%D0%9D%D0%B0%D1%83%D1%87%D0%BD%D1%8B%D0%B5_%D1%86%D0%B5%D0%BB%D0%B8_%D0%B8_%D0%B7%D0%B0%D0%B4%D0%B0%D1%87%D0%B8
Аноним 21/02/21 Вск 02:13:33 637197298
>>637073
Там не просто конвертация джоулей в антиматерь, тебе ее еще надо улавливать, сепарировать и удерживать нафиг. С удержанием самая большая сложность.

>>637074
Можно хранить ионы тащемта.

>>637136
>для того, что бы полететь к звездам, человечеству придется измениться
Согласен, и дело даже не в антиматерии.
Аноним 21/02/21 Вск 02:14:22 637198299
>>637178
>Там нужно дохуя топлива чтобы изменить орбиту большого астероида, может статься что доставляя это топливо в космос с земли, чтобы загрузить в чудесный изменитель орбит астероидов, ты потратишь больше денег, чем в итоге заработаешь разрабатывая астероид на орбите.
Так ты прямщас хочешь, а можно же за десять лет на ионниках гравитрактором стащить куда надо.
Аноним 21/02/21 Вск 02:23:51 637199300
Есть ли какая-то конструкционная причина почему Протоны в целом имеют 10% отказов в отличие от Союзов, например?
По идее же наоборот, должна быть надежность из-за простоты в гиперголиках, например. Союзы спичками поджигают, там-то по идее неравномерный поджиг в камерах, коих 20 маршевых, должен приводить к жопе чаще.
А почему-то у Протона не самая красивая статистика.
Аноним 21/02/21 Вск 03:26:54 637204301
>>637194
Алсо, есть ли какие-нибуд книги с фантазийные sci-fi или что-то такое, с упоминанием или описанием добычи на астероидах?
Аноним 21/02/21 Вск 04:13:19 637205302
6c1645694efc39f[...].jpg 22Кб, 271x450
271x450
2d6303d041d609d[...].jpg 82Кб, 680x680
680x680
DS306.jpg 72Кб, 576x325
576x325
>>637204
>или что-то такое, с упоминанием или описанием добычи на астероидах?
Аноним 21/02/21 Вск 04:23:28 637206303
школьник 13.jpg 65Кб, 604x403
604x403
>>637057
>Общество потребления до звезд не долетит.
Коммидебил, съеби уже в свой загон для умалишённых.
Аноним 21/02/21 Вск 04:27:35 637207304
>>637205
3 пикча это скрины из тибериумов?
Аноним 21/02/21 Вск 04:31:55 637210305
>>637206
Школьник тут только ты, раз мыслишь шаблоном "камунизьм плоха все камунисты дибилы", у тебя кругозор хлебушка обсмотревшегося инфлюенсеров, твое мнение стоит меньше мнения алгоритма гугла.
По делу ты ничего не сказал, даже не попытался факты противопоставить, какой-то аргумент привести.
Жалкое зрелище.

>>637057
Аватар хуета потому что хердостаний хуета. Из сайфач-кинца куда ни шло это интерстеллор, и тот со своей силой любви и 5д ЧД это пежня.
Аноним 21/02/21 Вск 09:42:31 637231306
>>637210
>силой любви и 5д ЧД
Ой, да, такая-то хуета! "Сила любви" блять, ну что за позорище, Нолан, нахуй, это режиссёр? Это тряпка! В чёрной дыре совсем уж бред бредской...
Да одна только сцена звонка домой в Луне (2112) сильнее пронимает, чем все эти нолановские библиотеки. Луна 2112 - вот это кино, разумный, элегантный минимализм и достоверность сюжетов.
Аноним 21/02/21 Вск 09:48:52 637234307
>>637154
Спасибо, анон. Очень классный арт ты тогда сделал, убедительный, наполнен реализмом: надуманную "романтику космических полётов" в сторону отставь, уважаемый - здесь среда чуждая, враждебная, изволь опробовать тяжёлый каждодневный труд!
Аноним 21/02/21 Вск 10:11:02 637236308
>>637075
Причиной, по которой Вселенная начала расширяться, стал вакуум, обладающий ненулевой энергией. Этот вакуум привел к инфляции Вселенной в самом начале ее жизни, а потом, после окончания инфляции, когда энергия вакуума перешла в рождение элементарных частиц, из которых сегодня состоит все вокруг, Вселенная просто по инерции продолжила расширяться. После инфляции она расширялась с замедлением, но 7 млрд лет назад она снова начала расширяться с ускорением. Причиной ускоренного расширения служит все тот же вакуум с ненулевой энергией, только плотность его энергии сегодня гораздо ниже, чем в период инфляции, поэтому темп расширения сегодня намного медленнее, чем был тогда.
Аноним 21/02/21 Вск 10:59:37 637240309
>>637231
Луна действительно отличный космический фильм, пусть и космос там лишь сеттинг.
Мне повезло что я его в кинотеатре смог в оригинале посмотреть, дубляжи в кинотеатрах нормальные, но мне просто приятнее слушать оригинал, самих актеров, не терять игру слов и отсылки. Минут десять потребовалось чтобы научиться не читать субтитры, отвлекали.
Но Интерстеллар тоже фильм лишь с сеттингом в космосе, он запросто мог бы переделаться на постапокалипсис и поездку в убежище 13 за ГЭКК, например.
Именно космически-ориентированных фильмов крайне мало.
Аполло-13 наверное самый космический среди всех.
Гравитация скорее всего тоже хорошо подходит.
Аноним 21/02/21 Вск 11:59:51 637260310
>>637240
А как же Салют-7,Пекло,Европа,Парадокс Кловерфилда
Аноним 21/02/21 Вск 12:02:54 637263311
>>637260
>Салют-7
Не смотрел, скорей всего тот же Аполло 13.
>Пекло
Это The Core почти что один-в-один.
>Европа
Не смотрел.
>Парадокс Кловерфилда
Не смотрел.
Аноним 21/02/21 Вск 12:44:24 637267312
>>637207
МЕРТВЫЙ ПРОБЕЛ же.
Аноним 21/02/21 Вск 12:53:12 637270313
image.png 873Кб, 1280x637
1280x637
>>637267
Я только первую часть прошёл давно, дальше не затянуло.
В тибериуме у Скринов подобная фигня была.
Аноним 21/02/21 Вск 13:15:26 637274314
>>637194
>Ну стыковка с вращающимся объектом уже не новость.
Это когда? На астероиде нету стыковочного узла. Он вращается и ничто ты с ним не сделаешь.
>А что не так с нагревом и охлаждением?
Ну ты как бы к нему должен прикоснуться, а он горячий, или холодный. А еще может взорваться если ты изменишь температуру его.
>Можно что-то похоже на пик1 соорудить.
Это хуйня какая-то для супермелких камней. Плюс ты его в ящик запихнёшь и он там по стенкам ебаться начнёт, порвёт это все еще, начнёт вытекать и сыпаться.

Аноним 21/02/21 Вск 13:18:20 637276315
>>637274
>Это когда? На астероиде нету стыковочного узла. Он вращается и ничто ты с ним не сделаешь.
Интерстелор.гиф блжад
>Ну ты как бы к нему должен прикоснуться, а он горячий, или холодный. А еще может взорваться если ты изменишь температуру его.
Не взорвется, не выдумывай.
>Это хуйня какая-то для супермелких камней. Плюс ты его в ящик запихнёшь и он там по стенкам ебаться начнёт, порвёт это все еще, начнёт вытекать и сыпаться.
Это да. Я в целом на твоей стороне, астероиды хуячить это пиздец как неудобно.
Аноним 21/02/21 Вск 13:19:57 637277316
>>637046
Расскажи, как ты видишь мир через 100лет?
Аноним 21/02/21 Вск 13:42:23 637282317
image 45Кб, 620x420
620x420
>>637274
>Это когда? На астероиде нету стыковочного узла. Он вращается и ничто ты с ним не сделаешь.
Ну хз, я же не про 600km гигачеда говорю,а про мелкие 15-20м околоземные, ну стыковка была СОЮЗ Т-13
https://ru.wikipedia.org/wiki/%D0%A1%D0%BE%D1%8E%D0%B7_%D0%A2-13#%D0%AD%D0%BA%D0%B8%D0%BF%D0%B0%D0%B6_%D0%B2%D0%BE%D0%B7%D0%B2%D1%80%D0%B0%D1%89%D0%B5%D0%BD%D0%B8%D1%8F

>Это хуйня какая-то для супермелких камней.
Ну так да) Для больших астероидов нужно будет уже на них организовывать производство добычу и тд, а вот мелкие можно ловить,и на орбиту луны отправлять и там разрабатывать.

>Плюс ты его в ящик запихнёшь и он там по стенкам ебаться начнёт, порвёт это все еще, начнёт вытекать и сыпаться.
Ну там же не будет мегаусилия какого-то постепенное продолжительное замедление и вывод на нужные орбиты.

Ну вот на пик1 солнечным лучем(или лазерным) испаряют астероид тем самым создавая тягу.
Аноним 21/02/21 Вск 13:44:07 637283318
>>637108
>Без конкуренции невозможно развитие
Схуяли? Без изменения среды скорее всего невозможно развитие. Да блять вся наша жизнь посторена на сотрудничестве митохондрии и эукариотической клетки. Нет в конкуренции нихуя хорошо, говно это ебаное.
Аноним 21/02/21 Вск 14:05:12 637285319
>>637283
Левачье, съеби на хуй из этого треда и вообще с этой борды. Пиздуй на порашу и там выкрикивай свои дебильные шариковские лозунги. Без конкуренции невозможна жизнь, так как конкуренция - ключевой механизм естественного отбора, его двигатель.
Аноним 21/02/21 Вск 15:01:40 637288320
>>637285
Давно большей хуйни не читал. Эволюции абсолютно похуй.
Аноним 21/02/21 Вск 15:05:38 637289321
>>637288
Ну раз тебе неприятно, то да, хуйня.
Аноним 21/02/21 Вск 15:09:28 637290322
74bbd49bb68bd10[...].jpg 438Кб, 1280x2033
1280x2033
Возможна ли на космическом лифте промежуточная станция между поверхностью Земли и ГСО?
Аноним 21/02/21 Вск 15:57:07 637296323
>>637288
Долбоеб, эволюция происходит за счет естественного отбора. Естественный отбор - мотор эволюции. А естественный отбор без конкуренции невозможен, без естественного отбора невозможна эволюция, без эволюции невозможна жизнь. Включай логику, придурок блядь.
Аноним 21/02/21 Вск 16:03:52 637297324
Ты когда одеваешься тепло в холодную погоду, с кем конкурируешь? Все изменения происходят для выживаемости в определенной среде и все. Вон павлины доконкурировали своими хвостами между собой, на радость хищников или рога, которые по лесу ходить мешают.
Аноним 21/02/21 Вск 16:08:16 637298325
>>637297
Когда ты одеваешься ты конкурируешь со средой , за выживание. Когда ты строишь ядерную бомбу, ты конкурируешь с другой страной за влияние или за выживание(в зависимости от накала отношений), мои и твои предки конкурировали за костный мозг оставленный хищниками съевшим мясо убитого животного. Павлины доконкурировали, до того что своими хвостами манят самочек ,а если ты выживаешь с таким хвостом, значит что-то в тебе есть - преимущество.

мимо другой анон
Аноним 21/02/21 Вск 16:14:26 637300326
>>637298
Когда ты читаешь двач ты конкурируешь с буквами. Когда ты вонюче срешь ты конкурируешь с воздухом в толчке. Когда ты дышит, конкурируешь с углекислым газом за выживаемость.
Я хоть что-нибудь могу сделать не конкурируя?
Аноним 21/02/21 Вск 16:17:40 637301327
>>637300
нет. ничего. везде конкуренция. ты пронизан ей. ткань мироздания есть ткань конкуренции с пространством-временем преисполнившись этой идей я стал бесконечно вечным преобразовав себя в энергию чистой как девственный взрыв суперновой и улетев покорять бесконечное пространство вселенной попутно решаю капчу на дваче.
Аноним 21/02/21 Вск 19:40:35 637323328
>>637321
Ну так я про тоже, твоя цель выжить, а не конкурировать с кем то. Если ты альфач - даешь пизды, что бы сожрать кого-то, если омежка - зарываешься под землю и сычуешь там, а если ты бета слон или бегемот, то тебе похуй все. Нет никакой конкуренции, есть только задача выжить и оставить свое потомство и ВСЕ. Нехуй тут плодить сущностей. А сотрудничества в природе хоть жопой жуй. В твоем теле только 10% клеток с твоим днк, остальные (понятно, что не все оставшиеся 80%) живут с тобой в сотрудничестве.
Аноним 21/02/21 Вск 19:42:27 637324329
>>637323
90%
не хотелось фификсить, но лучше мое одно, чем 3 чужих поста
Аноним 21/02/21 Вск 19:47:10 637328330
>>637323
> Ну так я про тоже, твоя цель выжить
Ну да. А для этого приходится конкурировать с другими.
Аноним 21/02/21 Вск 19:56:29 637329331
>>637328
Ну назови мне хоть один пример, кроме внутривидовой конкуренции (а мы уже вроде разобрались, что это хуйня полная и скорее вредит виду, чем приносит пользу), в котором я бы не смог, заменить "конкуренцию" на "выживаемость" и это впринципе никак не изменило бы смысл.
Аноним 21/02/21 Вск 19:57:50 637330332
>>637329
ветка борды - космос, космос производная от военных, война = конкуренция. Интернет был сделан военными, компьютеры сделаны военными, какие ещё вопросы?

мимо другой анон
Аноним 21/02/21 Вск 19:58:32 637331333
>>637329
> Ну назови мне хоть один пример, кроме внутривидовой конкуренции (а мы уже вроде разобрались, что это хуйня полная и скорее вредит виду, чем приносит пользу), в котором я бы не смог, заменить "конкуренцию" на "выживаемость" и это впринципе никак не изменило бы смысл.
Словоблуд и пустослов, придумывающий витиеватые описания, вместо использования подходящего по смыслу слова в лоб.

Таким левачки-коммибляди любят заниматься кста.
Аноним 21/02/21 Вск 19:58:51 637332334
>>637329
Ты можешь заменять что и как угодно, если тебе от этого легче, но конкуренция никуда не делась.
Аноним 21/02/21 Вск 20:07:45 637335335
>>637330
>>637331
>>637332
Ладно, я проиграл эту конкуренцию, останемся со своими конкурирующимися точками зрения, да и со временем не по-конкурируешь, завтра ехать в конкуренции на работу и там еще конкурировать надо.
Аноним 21/02/21 Вск 20:08:13 637336336
>>637329
Дегенерат, весь естественный отбор - это сплошная конкуренция между генами. Естественный отбор движет эволюцией. Следовательно, эволюцией движет конкуренция. Иди читай "Эгоистичный ген" Докинза и не неси хуйню, долбоебушка.
Аноним 21/02/21 Вск 20:11:36 637339337
1613927473934.png 179Кб, 413x395
413x395
>>637335
Ты, видимо, думал, что ворвёшься сюда и всех поставишь на место своим ахуенно важным мнением, но когда тебе сунули то, что выше твоего понимания, ты просто разорвал себе жопу от ванезапного понимания своей ущербности?
Аноним 21/02/21 Вск 20:50:50 637352338
7681bf7a1d24a93[...].png 9Кб, 950x154
950x154
>>637341
Забавно, что ты меня сбил своим этим постом >>637295 ,а этим постом опять вернул в колею. Я же первым своим постом >>637283 не отрицал наличие конкуренции, я говорил, что конкуренция говно, а сотрудничество - заебись. В итоге я скатился до того, что начал отрицать конкуренцию.
Привет тебе, чувак, >>637045
>Человеческий мозг - это очень совершенный компьютер.
Я тебе еще на этот пост хотел пару вебм сконвертировать из "Поля разума", но забил, даже пиком пруфану.
Бля, сорян, аноны, реально погнал, мозг конченная багованая хуйня.
Аноним 22/02/21 Пнд 03:18:20 637428339
Какова скорость звука в космосе в пределах Солнечной системы?
Аноним 22/02/21 Пнд 03:22:29 637429340
>>637428
в вакууме есть звук?О_О
Аноним 22/02/21 Пнд 03:28:19 637431341
>>637429
Ну да, а как же по твоему там ударные волны то образуются если скорости звука нет?
Аноним 22/02/21 Пнд 08:54:20 637453342
>>637428
null
Нет скорости звука.
Аноним 22/02/21 Пнд 11:11:45 637476343
Аноним 22/02/21 Пнд 19:30:35 637547344
>>637476

Какие ударные волны?
Ударная волна, это область воздуха с другим давлением.
Именно воздух и является источником передачи звука.
Аноним 22/02/21 Пнд 19:32:13 637548345
>>637199

Конструкционных причин нет. Это охуенно надежная ракета, одна из лучших в мире.
Проблема исключительно в культуре производства.
Аноним 22/02/21 Пнд 19:41:56 637552346
Screenshot20210[...].jpg 414Кб, 720x1520
720x1520
Астрономы, плиз ответьте, что это за хуета на Сохо?
Аноним 22/02/21 Пнд 19:44:46 637553347
>>636816

Способна развалить нейтроны на кварки, а при дальнейшем коллапсе, перестают существовать и кварки как таковые.
Как ты верно заметил, сингулярность - математический термин. А значит все, о чем мы рассуждаем - теория.
На самом деле, мы пока понятия не имеем, что находится за горизонтом событий, поскольку он - надежно закрывает от нас строение черной дыры.
Однако масса в 3 (например) солнечных, никуда не исчезает при коллапсе. Что из себя начинает представлять материя - одна из загадок.
И пока ответа на это нет.
Аноним 22/02/21 Пнд 19:48:20 637554348
>>637552
это летающая тарелка
Аноним 22/02/21 Пнд 19:56:41 637555349
>>637554
А если серьезно? Откуда такое свечение и вроде как уже второй день
Аноним 22/02/21 Пнд 19:59:45 637557350
>>637555
это звезда взрывается
Аноним 22/02/21 Пнд 21:37:22 637567351
bow-shock-LL-Or[...].jpg 35Кб, 800x667
800x667
>>637428
Tldr: 13-240 км/с.
Если понимаешь по английски, вот исчерпывающий ответ на этот вопрос:
https://physics.stackexchange.com/questions/162184/what-is-the-speed-of-sound-in-space/179057#179057
Если коротко, то межпланетная среда по большей части состоит из ионизированного, крайне разреженного и очень горячего газа.
Для плазмы, в отличие от нейтрального газа, можно выделить несколько типов волн, при распространении которых меняется плотность среды. Какие из них считать "звуком" каждый решает сам. Автор рассматривает ионно-звуковые волны как ближайший аналог звука в нейтральном газе. Скорость их распространения в зависимости от условий среды в межпланетном пространстве составляет 13-240 км/с.
Аноним 23/02/21 Втр 02:56:32 637760352
>>637547
>Какие ударные волны?
Например, от взрыва сверхновых.

>Ударная волна, это область воздуха с другим давлением.
Главное - это скорость звука в среде. Без скорости звука никаких ударных волн быть не может.
Аноним 23/02/21 Втр 02:57:05 637761353
>>637428
>Какова скорость звука в космосе в пределах Солнечной системы?

Примерно 10 км/c
Аноним 23/02/21 Втр 14:02:33 637834354
Увидим ли мы SSTO в ближайшие годы?
Аноним 23/02/21 Втр 14:27:26 637847355
>>637834
Ближайшие годы это сколько?
Аноним 23/02/21 Втр 19:54:22 637896356
>>637847
Лет десять-пятнадцать.
Аноним 23/02/21 Втр 20:07:15 637898357
>>637896
Не увидим. Разве что пиндосы решат весь свой военный бюджет в это влить.
Аноним 24/02/21 Срд 03:13:10 637952358
Посоны, читал я тут про варп двигатель, и вроде как сделал такой вывод - теоретически эта штука вполне реальна, но все упирается в то, что нужно слишком дохуя топлива/энергии чтобы заработало.
Поправьте меня плиз если неправильно сказал.
И еще второй вопрос про emDrive - слишком много противоречивой информации находил, где то пишут что это впринципе не должно работать, где то что работает заебись, а где то что работает но совсем чуть чуть.
Помогите нубу разобраться.
Аноним 24/02/21 Срд 03:24:55 637954359
>>637952
>Посоны, читал я тут про варп двигатель
Это фантастика и к науке не имеет никакого отношения.
Аноним 24/02/21 Срд 04:09:54 637960360
>>637834
>Увидим ли мы SSTO в ближайшие годы?
Вроде бы старшип без ПН может в SSTO, так что если допротипируют до орбитального прыжка - то увидим.

>>637952
>Посоны, читал я тут про варп двигатель, и вроде как сделал такой вывод - теоретически эта штука вполне реальна, но все упирается в то, что нужно слишком дохуя топлива/энергии чтобы заработало.
Всё упирается в отрицательную плотность энергии, а как её создать - хуй его знает. Надо или горизонты событий какой-то хитрой конфигурации мутить и фармить антиэнергию (не путать с антиматерией) излучением Хокинга, или устраивать какой-то ебический взрыв с падением плотности энергии ниже нуля во второй полуволне.
Запрета на такую отрицательную плотность нет, в какой-то мере она даже наблюдается в гравитационных волнах, но градиенты там слабенькие. Если каким-то образом скукожить отрицательную плотность в антисингулярность - то локальная скорость света вблизи неё будет расти (гравитационное замедление времени наоборот) вплоть до бесконечной. А если слепить из неё пузырь хитрожопой формы - получится этот ваш гипердрайв альбукерке.
>И еще второй вопрос про emDrive - слишком много противоречивой информации находил, где то пишут что это впринципе не должно работать, где то что работает заебись, а где то что работает но совсем чуть чуть.
Нихуя не ясно. По классической физике работать не должен, по всяким манятеориям волны-пилота и бурлящего вакуума - возможна тяга в десятки микроньютонов (что, впрочем, на порядки больше обычного фотонного фонарика).
На земле нормально экранировать всю хуйню не получается. Надо закинуть это ведро куда-нибудь на высокую орбиту, оставить там на пару лет и проверить, что изменилось, тогда будет конкретика. Китайцы вроде пробуют.




Аноним 24/02/21 Срд 06:31:16 637969361
>>637952
>И еще второй вопрос про emDrive - слишком много противоречивой информации находил, где то пишут что это впринципе не должно работать, где то что работает заебись, а где то что работает но совсем чуть чуть.
Все там понятно, эта хуйня не работала и не могла работать. Вся инфа на стенде от силы Ампера с проводов.
Аноним 24/02/21 Срд 10:33:26 637986362
>>637898
А нахуя он нужен. Вот та херня по типу Спирали уже и есиь SSTO:многоразовый, без обслуживания, быстрый реюз. Подвести на взлётную площадку новый бак-бачкок и состыковать *бак-космоплан-самолёт" дело нехитрое. ССТО хуйня с 2% ПН и не нужен.
Аноним 24/02/21 Срд 10:35:37 637987363
>>637986
>не нужен
Я и не спорю, ты постом ошибся.
Аноним 24/02/21 Срд 10:53:07 637989364
>>637986
>та херня по типу Спирали уже и есиь SSTO
Фактически всё же нет.
Аноним 24/02/21 Срд 11:43:05 637994365
>>637989
Так это исключительно потому что не построили.
Аноним 24/02/21 Срд 12:10:09 637998366
Аноним 24/02/21 Срд 12:27:18 638003367
>>637952
Это все ебучие фантазии, не более того.
Аноним 24/02/21 Срд 15:00:30 638020368
Короче я не понял, почему до сих пор не доставлен на Землю грунт с Марса или хотя бы с Фобоса?
Аноним 24/02/21 Срд 15:14:39 638021369
Аноним 24/02/21 Срд 16:40:59 638049370
>>638021
О какой тогда отправке человека на Марс можно вести речь, если даже ракетку со 100 грамами грунта до сих пор не могут пульнуть с оттуда?
Аноним 24/02/21 Срд 16:42:38 638050371
>>638049
Никто и не ведет речь об отправке человека на Марс в ближайшем будуйщем
Аноним 24/02/21 Срд 16:43:59 638051372
Два нубских вопроса
1) Почему ракетопланы не взлетели? Ведь можно просто добавить бак с окислителем, как делали на Старфайтерах ещё в 50-е.
2) Почему не взлетела космическая заправка? Когда к МКС пристыковывается условный челнок, потом грузовик доставляет топливо, этим топливом заправлется челнок и летит куда надо. Вместо того, чтобы тратить почти всё топливо на отрыв от Земли.
Аноним 24/02/21 Срд 16:50:22 638054373
>>638051
>1) Почему ракетопланы не взлетели? Ведь можно просто добавить бак с окислителем, как делали на Старфайтерах ещё в 50-е.
Самолет это максимум 1000 м/с и то на пределе возможностей, орбита это еще 7км/с помимо этого сверху. Тебе один хуй нужна ракета, а крылья не нужны.
Так что проще на ракетах.
>2) Почему не взлетела космическая заправка? Когда к МКС пристыковывается условный челнок, потом грузовик доставляет топливо, этим топливом заправлется челнок и летит куда надо. Вместо того, чтобы тратить почти всё топливо на отрыв от Земли.
Потому что некуда летать.
Аноним 24/02/21 Срд 18:51:46 638081374
>>638020

Охуенно сложный проект. Тебе по факту нужно 2 корабля. Один для путешествия в космосе и второй, для спуска и подьема назад. Лететь туда около полугода. Дальше или ждать 2 года на орбите (что бы сэкономить топливо), либо незамедлительно стартовать назад. Думаю, проще ждать 2 года, но там свои подводные камни.
Тебе нужно разогнаться до 2 космической, что бы оторваться от Земли. Потом затормозить до 1 космической около Марса, отправить туда полноценный космический аппарат. Допустим он возьмет пробу прямо там, где приземлиться. Потом ему надо взлететь. Придется преодолевать атмосферу и гравитационное поле Марса, которое выше в 2 раза чем у Луны.
Сколько все это будет - в душе не ебу. Много.
Все должно будет отработать чисто автоматически.
В принципе, думаю технологии позволяют реализовать такой проект.
Аноним 24/02/21 Срд 18:56:16 638083375
>>638049

Дальше картинок дело не пойдет. Амеры, конечно, что-то крякнули про 2030й год, Но нечто похожее я слышал про 2020й в 2010м.
Допускаю мысль, что возможно смогут облететь вокруг Марса (не понимаю, правда, нахуя), но, допустим, в рамках подготовки высадки, допускаю.
Без подготовленной базы на Марсе - ловить там нехуй. А значит, запасы воды, пищи, энергетика итд.
Вот когда на Марс полетят АМС с грузами, тогда можно будет надеятся на это. А пока - просто смотрим мультики.
Аноним 24/02/21 Срд 19:00:24 638084376
>>637994
Не, там же внешний бак отделяется, и довыведение на двигателях орбитального маневрирования - всё, уже не «единственная ступень».
Аноним 24/02/21 Срд 21:24:09 638097377
>>638020

Потому что доставка 100кг зонда с Марса потребует доставки примерно 5т груза на Марс. Очень дорого, проще туда лабораторию отправить.

>>638049

Когда звездолёт этот СпейсИксовский заработает то стоимость пуска приблизится к стоимости расходуемого топлива и отправка на Марс чего-то более тяжёлого, чем марсоход весом в тонну перестанет быть охуительно дорогим делом.
Аноним 24/02/21 Срд 21:27:27 638098378
>>637834

Нет, не увидим.
Когда есть возможность возвращать и сажать ступени то необходимость в SSTO отпадает.
Аноним 24/02/21 Срд 22:32:30 638101379
>>638097

Кстати, Фобос Грунт программа была.
Успешно сгорела над Тихим океаном.
А жаль.
Аноним 24/02/21 Срд 23:45:43 638114380
>>638097
>Когда звездолёт этот СпейсИксовский заработает то стоимость пуска приблизится к стоимости расходуемого топлива
Оно уже давно должно было так с флаконами произойти, а чето нихуя, они сколько стоили столько и стоят, Б/У ступени за 90% цены новой разве что.
Никакого колдунства илоний не предложит, т.к. наебать сопроматы и здравый смысл не может, от него чудес не надо ждать.
Аноним 25/02/21 Чтв 00:54:04 638123381
>>638101
Just as planned.
Если бы реально в успех верили, уже повторили бы.
Аноним 25/02/21 Чтв 08:01:42 638149382
>>638114
У флаконов вторая ступень в утиль идёт. И система управления со всей поебенью в утиль идёт. Поэтому экономия хоть и есть, но не радикально огромная.
Старшип - это будет уже фулл реюз, причём в отличие от шаттла со сверхдешёвыми расходниками.
Аноним 25/02/21 Чтв 08:30:51 638154383
>>638149
>Старшип - это будет уже фулл реюз
Точно так же как будет реюз второй ступени? Точно так же как реюз первой ступени свел ее стоимость к стоимости топлива? Точно так же как Red Dragon высадился на марсе?
Сколько раз надо поменять/отменить планы, чтобы вы перестали слепо верить новым?
Пока что Старшип настолько же успешный и прорывной как скакун Безоса и спейсшип Бренсона - обкатываемые прототипы ничего нового не добавившие.
Аноним 25/02/21 Чтв 09:47:06 638159384
>>638154
Старшип это двигатель в первую очередь. По многократным включениям он уже вывозит. По УИ вывозит. По тяге вывозит. Приземление доработают, Ф9 тоже не с первого раза посадить получилось.
Единственное, что пока представляет неясность - это тепловые процессы при реентри. Нихуя неясно, что там с плитками, и сколько их надо. Если этот вопрос победят - во всём остальном старшип не выглядит невозможным.

Red Dragon можно сделать хоть прямо сейчас, прикрутив CrewDragon на фалконхеви и загрузив под завязку картохой. Просто он стал не нужен. Это наивная флаговтык-миссия, которая становится не нужна, когда ты уже в состоянии сделать полноценный рейсовый автобус на 100+ тонн.
Безос пока ничего революционно нового не рожает, а тупо пытается воспроизвести Ф9 побольше и потолще, как до этого воспроизвёл толстый грассхопер ака НьюШепард. Повторюшка дядя хрюшка из помойного ведра. Никаких запусков по цене топлива там никогда не предполагалось.
Спейсшип Бренсона никогда не задумывался для орбитальных полётов и не имеет по сути отношения к космосу вообще. В лучшем случае из него можно слепить мажорский трансатлантический лайнер в 2-3 раза быстрее конкорда.
Аноним 25/02/21 Чтв 10:34:26 638161385
>>638154
>обкатываемые прототипы ничего нового не добавившие.

Раптор, стальной корпус, посадка, аэродинамические рули. Это всё давно используется, конечно. Раптор, конечно, охуенен. Удобное топливо, высочайший УИ и тяга к весу, многократное включение, дросселируемость.
Аноним 25/02/21 Чтв 10:35:11 638162386
>>638114
>Оно уже давно должно было так с флаконами произойти

Выдумки.

Аноним 25/02/21 Чтв 10:38:08 638163387
>>638159
Я не понимаю, зачем ты эту простыню вытащил, будто обиделся на скептицизм.
Многократные движки были уже давно и летали десятки раз, муск тут тоже опять не первопроходец.
>Единственное, что пока представляет неясность - это тепловые процессы при реентри. Нихуя неясно, что там с плитками, и сколько их надо. Если этот вопрос победят - во всём остальном старшип не выглядит невозможным.
Не если, а когда победят. СС не является невозможным, в нем нет ничего экстраординарного, о том-то и речь.

>>638161
Другими словами - спейс шаттл как он должен быть без вмешательства воене требований, то что в семидесятые и могли бы сделать.

>>638162
Толсто.
Аноним 25/02/21 Чтв 13:32:19 638194388
У меня появилась новая идея насчёт повышения альбедо Земли. Для охлаждения мексиканского залива можно было бы, например, заливать его нефтью/маслом, а затем посыпать это алюминиевой крошкой. Сработает ли эта схема по созданию собственного алюминиевого зеркала для морей и океанов?
Аноним 25/02/21 Чтв 13:52:51 638201389
image.png 150Кб, 280x204
280x204
>>638194
Ты забыл про бульон. Поливать мексиканский залив нефтью и засыпать люминем приведет лишь к новому стремному способу доставки нефти и алюминия в британию и скандинавию.
Аноним 25/02/21 Чтв 13:53:18 638202390
Аноним 25/02/21 Чтв 14:06:38 638209391
А что если у пространства есть своя плотность? И чем она выше, тем медленнее протекают физические процессы. Тоесть чем выше у объекта гравитация, чёрная дыра например, тем больше вокруг неё притянутого и сжатого пространства.

А если при помощи антигравитационной машины растянуть область пространства, то можно будет перемещаться свыше скорости света.
Аноним 25/02/21 Чтв 14:10:24 638211392
Шанс растянуть пространство антигравитацией примерно такой же, как и молитвой.
Аноним 25/02/21 Чтв 14:19:52 638214393
image.png 1839Кб, 2842x591
2842x591
Аноним 25/02/21 Чтв 15:14:46 638239394
Что случится с планетами нашей системы, когда Солнце станет белым карликом? Каменных сожрёт красный гигант, это понятно. А газовые? Просто разлетятся по космосу?
Аноним 25/02/21 Чтв 15:23:04 638243395
>>638239
Никуда они не разлетятся, так же и продолжат вращаться вокруг белого карлика. Масса у него будет почти такая же, как у Солнца.
Аноним 25/02/21 Чтв 15:24:03 638244396
>>638209
Иди поучи специальную и общую теорию относительности и не неси хуйню.
Аноним 25/02/21 Чтв 15:42:58 638254397
>>638239

Ну судя по расчетам, красный гигант гарантированно сожрет Венеру и, с большой вероятностью, Землю.
С Юпитером ничо не случится, он останется газовым гигантом.
А вот Сатурн, окажется в обитаемой зоне, и как знать, какие подводные могут быть при этом.
Аноним 25/02/21 Чтв 16:08:03 638259398
>>638254
Нихуя. Венеру не сожрёт, Меркурий даже не сожрёт. Смотри параметры Арктура, звезды с массой ≈1 солнечную. У него радиус около 0,1 а.е. и светимость 170 солнечных.
Так что вообще нихуя не будет: Меркурий на 0,28 а.е. останется, Венера останется, а Титан станет как Венера (1,7 земной инсоляции +30-40 градусов парникового эффекта от метана + ещё хуй знает сколько от паров воды). Неа, Титан всё, планета-пиздец, планета-жизнинет.
Аноним 25/02/21 Чтв 16:15:05 638261399
>>638259
А вот Тритон! 1/6 земной инсоляции + толстая атмосфера с метаном...хмм...
Аноним 25/02/21 Чтв 16:16:58 638262400
>>638261
И даже Титания с Обероном могут на миллион-два эдак лет закутаться в атмосферу CO2.
Аноним 25/02/21 Чтв 16:26:51 638263401
Что такое великий аттрактор?
Аноним 25/02/21 Чтв 16:30:09 638264402
>>638201
В этом и вся идея. Если сделать Гольфстрим холодным, то арктические льды со снегом в среднем в год могут запросто начать занимать пространство на десятки процентов больше обычного, что значительно охладит Землю. Для предотвращения утекания масла можно сделать специальную плывучую плавучую перегородку между Флоридой и Кубой, что и делают при обычных разливах.
Аноним 25/02/21 Чтв 16:34:59 638265403
>>638244
А что не так? Чем массивней тело, тем больше на него падает пространства и оно сильней сжимается. Например километр сжимается до метра, и этот сжатый метр свет будет лететь столько же времени, как и километр вне зоны гравитации.
>>638214
Еслиб я ещё инглиш знал.
Аноним 25/02/21 Чтв 16:54:54 638268404
>>638263
эта ос на который крутитса армения
Аноним 25/02/21 Чтв 17:24:35 638271405
>>638265
Блядь, пространство не "сжимается". Что за ебанутый термин? В ОТО нет такой терминологии. Там пространство-время искривляется под действием массы, а не сжимается. Искривление пространства-времени - это и есть гравитация.
Аноним 25/02/21 Чтв 17:25:08 638272406
>>638269
1. Возможно, в будущем человечество это сможет как-то использовать для спасения своей задницы/терраформинга других миров.
2. Просто интересные размышления. Превращение Земли в ледяной снежок даже на небольшое время приведёт к практически полному обнулению жизни. И это, наверное, единственный вполне легальный способ устроить настоящий апокалипсис на Земле.
Аноним 25/02/21 Чтв 17:26:15 638273407
25/02/21 Чтв 18:20:18 638281408
>>638259
С титана скорее всего вся атмосфера испарится
Аноним 25/02/21 Чтв 18:25:29 638282409
>>638281
Неа. У красного гиганта темпа низкая, мало ультрафиолета. Так что перегрева экзобазы не будет, а без этого, при T=400-500К азот не уйдёт, там скорость молекул 500-550 м/с, в пределах возможностей Титана.
Аноним 26/02/21 Птн 21:43:57 638440410
Может ли быть большой и при этом относительно холодный гигант, карлан или даже с наше солнце, объект, являющийся по всем параметрам звездой, при этом такой, что его поверхность выходить за пределы воображаемой линии границы предела Роша?
Могут ли у такой "рыхлой" звезды, быть ультраблизко планеты?
Могут ли они быть обитаемыми?
Аноним 26/02/21 Птн 22:02:30 638441411
>>638440
>Может ли быть большой и при этом относительно холодный гигант, карлан или даже с наше солнце, объект, являющийся по всем параметрам звездой, при этом такой, что его поверхность выходить за пределы воображаемой линии границы предела Роша?
Да.
>Могут ли у такой "рыхлой" звезды, быть ультраблизко планеты?
Да.
>Могут ли они быть обитаемыми?
Нет.
Это последняя стадия звездной эволюции.
Аноним 27/02/21 Суб 07:20:44 638466412
Если завести тысячу тонн биомассы-компоста с земли с земными же микробами на какой-нибудь марс/титан/европу есть ли шанс что бактерии в компосте рано или поздно приспособятся к местным условиям из-за эволюции?
Аноним 27/02/21 Суб 08:28:33 638471413
Аноним 27/02/21 Суб 17:59:53 638508414
В прошлом треде спрашивал про орбитальный велосипед. А если ноги отрубить?
Аноним 27/02/21 Суб 18:29:24 638512415
>>638508
Тогда никаких тебе велосипедов.
Аноним 27/02/21 Суб 19:01:36 638517416
Что-нибудь прорывное обещают в ближайшие 50 лет, чтоб за ни дорого туристом хотя бы на луну можно будет слетать? а лучше на Титан какой
Аноним 27/02/21 Суб 19:08:21 638520417
>>638517
Нет. Космонавтика это большей частью до сих пор социополитическая хуйня, так что даже туризм будет едва развит.
Технологически международная кооперация по постройке пусковой петли могла бы удешевить выход в космос на порядок, но ни этой кооперации, ни петли не предвидится.
Аноним 28/02/21 Вск 03:49:50 638567418
>>638520
>Космонавтика это большей частью до сих пор социополитическая хуйня, так что даже туризм будет едва развит.

На 100% Советский Союз в свое время подложил всему человечеству огромную свинью когда запустил челика в космос и зафорсил идиотскую идею полета человека в космос.
Аноним 28/02/21 Вск 06:46:24 638580419
Почему SETI ищут маяки и сами маякуют в радиодиапазоне, а не в отпике? Проще же вывести на орбиту небольшой лазер на пару десятков киловатт и прицельно маяковать им.
Аноним 28/02/21 Вск 08:20:09 638585420
>>638580
Небольшой прицельно неполучится из-за расхождения. Для межзвёздной связи надо огромную хуёвину строить на сотню метров апертуры или распределённый массив какой-то.
Аноним 28/02/21 Вск 08:30:13 638586421
Я заметил, что кидая мяч в сторону он отталкивает меня немного, а если кинуть гантелю то оттолкнет сильно. Что если это использовать для движения, только вместо шарика с массой будет газ?
Аноним 28/02/21 Вск 09:15:17 638593422
>>638586
>Я заметил, что кидая мяч в сторону он отталкивает меня немного
Таблетки прими, наркоман.
Аноним 28/02/21 Вск 10:04:15 638602423
>>638586
Да ну, бред, газ же легкий. Предлагаю вместо этого использовать раствор урана, подавать по боровым трубкам, чтобы он начинал реагировать только потом и расплавом толкал тебя
Аноним 28/02/21 Вск 10:29:30 638603424
>>638585
>из-за расхождения.
а какое расхождение у лазерного луча на расстоянии например 10 парсек?
Аноним 28/02/21 Вск 11:18:27 638611425
e7500ba6ee13d80[...].jpg 57Кб, 600x628
600x628
Аноним 28/02/21 Вск 11:57:39 638616426
>>638611
Сука, ну вот что ты наделал. Теперь пересмотреть захотелось.
звук набора земных звёздных врат один из лучших сайфайных звуков, имхо
Аноним 28/02/21 Вск 12:44:41 638626427
>>638567
Опять хохлы говна всему человечеству в штаны наложили.
Аноним 28/02/21 Вск 13:10:06 638631428
Аноним 28/02/21 Вск 14:51:09 638653429
>>638466
Если действовать чуть менее топорными методами, то да. Но на Титане засевать нужно подземный океан, а не поверхность. На поверхности все вымрут на 100% из 100, эволюции не будет
Аноним 28/02/21 Вск 15:43:49 638656430
Когда первый полет вертолетика на Марсе?
Аноним 28/02/21 Вск 16:29:42 638661431
>>638656
>вертолетика на Марсе?
А он планировался? Там же атмосфера супер разряженная.
Аноним 28/02/21 Вск 16:36:21 638662432
>>638661
>А он планировался?
Его доставили же с марсоходом вместе.
Аноним 28/02/21 Вск 16:55:48 638663433
>>638662
Не взлетит, там слишком разряженныая атмосфера, идиоты, лучше бы чего полезного взяли.
Аноним 28/02/21 Вск 16:58:19 638664434
>>638663
>спейчар считает себя умнее инженеров наса
Кек, классека
Аноним 28/02/21 Вск 18:22:24 638667435
001.jpg 363Кб, 2477x1393
2477x1393
>>638663
>Не взлетит, там слишком разряженныая атмосфера, идиоты, лучше бы чего полезного взяли.
- Тяга пропорциональна первой степени плотности.
- Квадрат тяги пропорционален кубу мощности.
- Сила тяжести на Марсе 0.378 от земной.
- Плотность атмосферы у поверхности около 0.020 кг/м³ (1.67% земной).

Итого идентичному земному вертолётику на Марсе требуется в (0.378)^1.5/0.0167 = 13.9 раз больше мощности. Дохуя, но не запредельно. Если ободрать с него всё лишнее и повысить массовое совершенство в 2 раза, то получаем 13.9х0.5^1.5 = 4.9, уже более чем реальная цифра.

Далее, любой вертолёт можно запитать от сколь угодно малой мощности, увеличивая диаметр ротора. Увеличение в 2 раза даёт увеличение площади в 4 раза, снижение требуемой скорости струи в 2 раза и снижение мощности в 8 раз. Т.е. нашему вертолётику нужно нарастить ротор всего в 1.7 раз.

Теперь смотрим на Ingenuity. Минималистичная конструкция, увакованная в крошечный кубик с гипертрофированным ротором. В общем, ничего необычного.
Аноним 28/02/21 Вск 19:09:42 638675436
>>638667
А у нас ведь мощность тратится на преодоление лопастями сопротивления среды. А сопротивление среды пропорционально давлению, азаза. В итоге пропорциональность давлению со стороны тяги и со стороны мощности сократятся, и мы получим лишь снижение требуемой мощности из-за меньшей гравитации.
Аноним 28/02/21 Вск 19:30:33 638676437
>>638675
Аэродинамическое трение лопастей об среду это копейки по сравнению с тем, что тратится на ускорение струи вниз. В этом смысле кпд винта приближается к 100%. А проблему паразитной раскрутки соосная схема решает наиболее эффективно.
Аноним 28/02/21 Вск 19:43:18 638679438
>>638676
"Тратится на ускорение струи вниз" - это тоже описывается как аэродинамическое сопротивление, у него даже название есть, "индуктивное сопротивление" (мой внутренний электронщик кринжует). Но с ним все намного сложнее. С другой стороны, максимальное аэродинамическое качество крыла достигается, когда индуктивное сопротивление равно сопротивлению, не связанному с образованием подъемной силы, то есть в правильно спроектированном девайсе они соизмеримы.
Аноним 28/02/21 Вск 20:56:58 638690439
001.png 7Кб, 490x123
490x123
>>638679
Нет, ты всё перепутал. Есть "полная аэродинамическая сила". Она состоит из двух вредных составляющих и одной полезной:

1. Ускорение струи вниз. Каждый самолёт это немного вертолёт, а каждый вертолёт немного ракета. Подъёмная сила всегда создаётся за счёт отбрасывания массы воздуха вниз. Тяга равна (m/t)U, полезная мощность (m/t)U²/2. Чем ускорять струю - совершенно похуй, хоть ионным ветром. Важно лишь количество кг в секунду и скорость этой струи.

2. Лобовое сопротивление, оно же драг. Это сопротивление при нулевой подъёмной силе, потребное для расталкивания молекул, не связанное с ускорением их в одну сторону. Оно есть и у кирпича, и у шарика. У тонкого крыла оно тоже есть, но по сравнению с подъёмной силой очень мало. Буквально проценты. Считается по пикрелейтед формуле с эзотерическим коэффициентом Cf, который надо так примерно прочуствовать.

3. Индуктивное сопротивление. Это когда воздух пытается съебаться из градиента давлений и начинает перетекать из-под крыла наверх. С края крыла при этом срывается вихрь, он же жгут, который жрёт мощность на вращение. Считается очень заёбно, ибо отожранная мощность очень дохуя от всего зависит, её даже рекуперировать можно и использовать для дополнительной подъёмной силы.
Аноним 28/02/21 Вск 21:13:41 638693440
>>638667
Если вертолеты так легко усиливаются, то давно бы уже на эверест с вертолета высаживались вместо того чтобы умирать по дороге сотнями.
Аноним 28/02/21 Вск 21:15:20 638694441
>>635688
>2. Если бы угол входа челябинского метеорита был не 18 градусов, а 90, то каковы были бы последствия?
Мощность взрыва, произошедшего в момент входа метеорита в плотные слои атмосферы над Челябинской областью, по оценкам NASA составила от 300 до 500 килотонн, что примерно в двадцать раз превосходит мощность атомной бомбы, сброшенной на Хиросиму.
Аноним 28/02/21 Вск 21:29:37 638698442
Аноним 28/02/21 Вск 21:34:36 638700443
>>638693
>вместо того чтобы умирать по дороге сотнями.
Так для этого и идут же. "Будь мужиком блеать", вот это вот всё. А максимальный потолок вертолёта намного выше, чем Эверест.
Аноним 28/02/21 Вск 21:49:30 638701444
>>638694
Ясно-понятно, Челябинск такую мелочь не заметил бы.
Аноним 01/03/21 Пнд 02:59:46 638736445
002.png 18Кб, 868x184
868x184
>>638693
Они не усиливаются (это не покемоны блядь), а оптимизируются под задачу, гипертрофируя одни фичи в ущерб другим. Вот так, например, выглядит вертолёт, способный поднять человека в воздух за счёт его собственных мускульных сил:
https://www.youtube.com/watch?v=syJq10EQkog

>>638698
Нет не противоречит, просто термин по-ёбнутому написан. И бернуллиевая интерпретация подъёмной силы тоже по-ёбнутому сформулирована, из-за чего копья до сих пор ломают, а отдельные кулибины пытаются взлететь, вращая диск.
Да, драг действительно lift-induced, но не из-за природы драга, а из-за побочного эффекта драга в виде создаваемого градиента давлений. Если делать крыло бесконечного удлинения, аппроксимируя задачу до двухмерной - никаких вихрей там не будет, а Fi будет нулевой. А подъёмная сила будет.
Можно и в трёхмерном пространстве эту хуйню обнулить на обратной стреловидности - именно через этот фокус там аэродинамическое сопротивление и снижается.
Аноним 01/03/21 Пнд 06:31:58 638745446
>>638736
>Если делать крыло бесконечного удлинения, аппроксимируя задачу до двухмерной - никаких вихрей там не будет, а Fi будет нулевой. А подъёмная сила будет.
Если ты делаешь удлинение λ бесконечным, а подъёмную силу Y конечной, то каждый погонный метр крыла у тебя производит неотличимое 0Н количество подъёмной силы.
Аноним 01/03/21 Пнд 07:07:35 638746447
>>638745
Удлинение это не длина, а отношение размаха (расстояния между вершинами) к хорде.
Аноним 01/03/21 Пнд 07:21:09 638748448
>>638746
То есть у тебя хорда является бесконечно малой величиной, что ли?
Аноним 01/03/21 Пнд 07:29:19 638749449
>>638736
>вертолёт, способный поднять человека в воздух за счёт его собственных мускульных сил:
Это невозможно же. Человек способен выдавать 100 ватт, а весит 1000 ньютонов, то есть 100 н-м-с выдает всего лишь, в 10 раз меньше чтобы себя поднять.
Аноним 01/03/21 Пнд 07:38:15 638750450
>>638749
Ты только что доказал невозможность подтягиваний на турничках.
Аноним 01/03/21 Пнд 07:48:15 638752451
225-22551529027[...].png 185Кб, 820x883
820x883
Аноним 01/03/21 Пнд 08:30:03 638755452
image.png 489Кб, 500x598
500x598
>>638750
А я и не могу подтягиваться...
Аноним 01/03/21 Пнд 08:55:35 638756453
Какие есть возможные технологии или способы транспортировки астероидов?
Аноним 01/03/21 Пнд 08:56:45 638757454
>>638750
Ты только что доказал возможность черепах летать
Аноним 01/03/21 Пнд 09:28:16 638758455
>>638756
Ракетный двигатель, обмазывание фольгой, гравитационный трактор.
Аноним 01/03/21 Пнд 09:29:02 638759456
image.png 105Кб, 250x188
250x188
>>638757
Нет ничего необычного в летающих черепах, я видел это в Думе.
Аноним 01/03/21 Пнд 10:57:19 638767457
Аноним 01/03/21 Пнд 14:34:05 638797458
Скажите честно, в 20 лет уже поздно в космос вкатываться?
Аноним 01/03/21 Пнд 14:37:39 638798459
Аноним 01/03/21 Пнд 15:16:31 638804460
>>638693
На эверест лезут ради процесса, а не просто что бы наверху посидеть.
Аноним 01/03/21 Пнд 15:25:38 638806461
>>638748
Типа того, такой гипертрофированный планер. Хотя и это не обязательно. Самолёт, летящий между двумя идеально прилегающими стенками, будет обладать де-факто бесконечным удлинением крыла, ибо вершин крыла у него де-факто нет, а поток становится одинаковым во всех поперечных сечениях.

>>638749
>видеопруф
>Это невозможно же.
Ебал я постиронию эту.
Аноним 02/03/21 Втр 12:54:29 638928462
Тэкс, наркоманский вопрос. Если мы смотрим в космос, то видим прошлое вселенной. А возможно ли где-то увидеть еще молодое наше Солнце? Скажем где-то на расстоянии 4 млрд. световых лет?
Аноним 02/03/21 Втр 13:10:18 638930463
Аноним 02/03/21 Втр 13:10:44 638931464
>>638928
Где ты собрался смотерть на наше солнце на растоянии 4 млрд. световых лет
Аноним 02/03/21 Втр 13:11:32 638932465
>>638931
А че он-то лично сразу?
Аноним 02/03/21 Втр 13:13:06 638933466
Аноним 02/03/21 Втр 13:14:57 638934467
>>638933
Люк Скайуокер, например, 4 гигагода это достаточно далекая-далекая галактика чтобы считаться далекой-далекой.
Аноним 02/03/21 Втр 13:22:45 638935468
>>638931
С Земли собрался. 4 млрд лет назад Солнце же не тут было. Вот откуда прилетело, в ту сторону и смотреть буду.
Аноним 02/03/21 Втр 13:39:18 638940469
>>638931

Можно, для наблюдения с любого обьекта, находящегося на расстонии 4 миллиардов св. лет, наше солнце только только зажглось.
Аноним 02/03/21 Втр 13:45:26 638942470
>>638935
Там ты его не увидишь, свет оттуда улетел почти на 4 миллиарда светолет во все стороны.
Аноним 02/03/21 Втр 16:38:12 638958471
>>638931
>Где ты собрался смотерть на наше солнце на растоянии 4 млрд. световых лет
Предположим, в отдалённой галактике у инопланеТЯНки есть косметичка, а в косметичке зеркало. Надо поймать момент, когда она начнёт красить своё ебло, вытащит косметичку, и зеркало окажется сориентировано по нормали точно на солнце.
Аноним 02/03/21 Втр 17:07:01 638959472
>>638611
И, как тогда эту штуку создать? Двач же на острие науки.
Аноним 02/03/21 Втр 17:49:48 638964473
>>638959
В казахстане такое уже делают
Аноним 02/03/21 Втр 22:35:33 639005474
>>638964
Сукаблять, что вспомнил у нас в подъезде была нарисована червоточину с формулами даже какими-то накуй. Лет 15 назад было.
Аноним 03/03/21 Срд 11:21:12 639068475
image.png 147Кб, 1200x739
1200x739
Можем ли мы построить пусковую петлю с современными технологиями?
Аноним 03/03/21 Срд 11:31:42 639071476
image.png 92Кб, 1092x1037
1092x1037
>>639068
Нет. Для этого нужна кооперация и хорошие финансовые и трудовые вложения.
Аноним 03/03/21 Срд 11:35:53 639072477
image.png 63Кб, 400x204
400x204
>>639071
Там же всего 30 млрд нужно, даже если понадобится 100 это все равно будет в два раза меньше чем на шаттлы потратили. Пиндосы себе без проблем столько напечатать могут.
Аноним 03/03/21 Срд 11:40:46 639073478
image.png 114Кб, 500x382
500x382
Аноним 03/03/21 Срд 11:42:02 639074479
image.png 63Кб, 265x255
265x255
Аноним 03/03/21 Срд 12:36:44 639084480
>>639072
30 млрд по оптимистичным, ни на чём не основанным оценкам. Ключевых технологий (масс драйверы, сверхдлинные деформируемые трубы, гигантские роторы) либо вовсе не существует, либо они находятся на очень низком уровне технической готовности.
Аноним 03/03/21 Срд 14:33:10 639096481
>>639084
Ну так хуячить надо.
Уран тоже не сразу обогащался, а теперь мы живем в мире который потенциально может быть полностью электрифицирован экологически чистой ядерной энергии, если бы не некоторые дегенераты.
Аноним 03/03/21 Срд 17:02:05 639127482
>>639096
Это действительно другое. Урановые реакторы работают просто как стержни кипятильников, опущенные в воду, проверить принцип их работы и ставить эксперименты можно на масштабе ну пусть не спичечного коробка, но хотя бы на простом столе, как американские диды делали.

Пусковую петлю можно сравнить скорее с термоядом с магнитным удержанием плазмы — где физические принципы очевидно верны, но минимальная работающая модель требует целого НИИ для постройки, а на первую модель с положительным выходом надо собирать деньги всем миром. Не думаю, что натурный эксперимент с петлёй длиной менее километров десяти и радиусом роторов в хотя бы сотню-другую метров вообще можно поставить — труба меньшего размера тупо не согнётся, да и заметно ее ускорить не выйдет.

С такой большой минимальной ценой захода и неясными перспективами сразу понятно, почему не находится желающих вложиться в проект.
Аноним 03/03/21 Срд 17:11:40 639130483
>>639127
>а на первую модель с положительным выходом надо собирать деньги всем миром
В этом-то и проблема.
Я вообще удивлен, что ИТЭР строится.
03/03/21 Срд 17:58:06 639143484
>>635533 (OP)
Сап, аноны. Есть такой вопрос: если Луна всегда повернута одной стороной к Земле из-за приливных сил, то почему Земля не повернута всё время к нашему родному солнцу одной стороной? Разве здесь не действуют те же эффекты и те же силы?
Аноним 03/03/21 Срд 18:01:42 639145485
>>639143
Далековато от слонца чтобы успеть прекратить вращаться за 4 гигагода. Приливные силы тем сильнее чем ближе к объекту.
Аноним 03/03/21 Срд 20:08:37 639195486
>>639143
Действуют те же эффекты и силы, но их не достаточно, солнце даже меркурий повернуть к лесу задом не может.
Аноним 03/03/21 Срд 20:10:43 639197487
>>639068
Почитал, годная хуйня. Ну неужели ни у кого после этого нет желания раскулачить кабанчиков и выкинуть их с Земли этой петлей? Эх, такие проекты простаивают, только потому что это не принесет сверхприбыли кабанчику здесь и сейчас.
Аноним 03/03/21 Срд 20:20:37 639200488
>>639197
Возникает, но как показывает практика после раскулачиваний становится ещё хуже.
Аноним 03/03/21 Срд 21:03:54 639219489
85c12822525a377[...].png 14Кб, 781x126
781x126
>>639200
Как показывает практика, кожанные мешки вообще больше хуйни делают, чем пользы. Мы же применем современные технологии, а не обезьянье хуемерство.
Аноним 04/03/21 Чтв 23:14:55 639869490
15790091623500.jpg 122Кб, 900x1200
900x1200
Может ли быть планета как-Земли, которая обладает достаточно массивной как-Луной, которая была бы как Луна у Земли, но при этом такая как-Земля находилась бы в приливном захвате к местной звезде, даже несмотря на вращение как-Луны?
Аноним 05/03/21 Птн 00:20:25 639885491
Если бы в соседней системе на орбите висел телескоп кеплер, то он бы записал наш марс и венеру в список экзопланет?
Аноним 05/03/21 Птн 00:49:48 639887492
>>639072
Сразу умножай на десять.
Нет ни одной технологии необходимой для петли, это просто концепт.
На планете нет кабанчика который бы это потянул.
Вот как это делается, сначала собираются учёные и описывают проект поступенчато, указывая какие технологии уже есть, какие необходимо доработать, а каких нет вообще. Это длится пару лет, комиссия международная изначально. Одна страна петлю не вытянет.
Далее собираются учёные, которые больше политики и продавливают в конкуренции с десятками иных проектов петлю. По сути их задача доказать правительствам ведущих стран что без петли всему пизда.
Далее собираются политики, учёные, финансисты и расчерчивают дорожную карту стройки и разработок.
Дальше или через 20 лет и 300 миллиардов баксов у тебя будет петля, или через пять лет и 30 миллиардов долларов проект закроют с рещолюцией невыполнимо.

Модельно, можешь посмотреть на чем основывалось решение строить гравитационные телескопы и как их строили.
Аноним 05/03/21 Птн 07:23:40 639905493
>>639885
И Землю тоже.
Но в первую очередь Юпитер.
Аноним 05/03/21 Птн 10:34:42 639922494
>>638806
>>видеопруф
>>Это невозможно же.
>Ебал я постиронию эту.
Снято в Голливуде.
Аноним 05/03/21 Птн 16:51:55 639964495
У Нептуна есть твёрдая поверхность? На схемах нарисована "мантия из льда и силикатов", но по классификации Нептун всё равно газовый гигант. Или там просто чёткой границы между мантией и сжимающейся до состояния сверхкритической жидкости атмосферы нет?
Аноним 05/03/21 Птн 18:01:09 639974496
>>639964
Нет там никакого льда. Каждый, кто делает такие схемы, должен писать длинное примечание, что понимается под словом "лёд". Иначе он позёр, надменное снобское чмо, мудак-обмудок. Для людей делаются схемы, ДЛЯ ЛЮДЕЙ, тех людей у которых "лёд' это водяной холодный твёрдый лёд, а не светящаяся от белого каления густая жидкость. По факту, что Уран-Нептун, что Сатурн-Юпитер суть океанические жидкие планеты, не газоые. На больших это океан водорода, на маленьких - грязной воды.
Аноним 05/03/21 Птн 18:06:07 639977497
image.png 1368Кб, 2560x2160
2560x2160
Аноним 05/03/21 Птн 18:38:47 639980498
Аноним 05/03/21 Птн 19:04:29 639982499
16124099516261.jpg 445Кб, 524x1476
524x1476
Может ли быть на микропланете типа Плутона, толстый, хотя бы в половину приемлимый для землянина по давлению слой атмосферы, который бы не сдувало?
Такое возможно, если нет солнечного ветра, значит, такая планета должна быть далеко, а значит, от солнца там энергии тепла и света будет мало. А как сделать там тепло и светло, чтобы жизнь зиждилась? Может, как-то присобачить сильно магнитное поле?
Аноним 05/03/21 Птн 19:10:35 639984500
>>639982
Проблема атмосферы у мелкохуйни не в том что ее сдувает, а в том что ее молекулы улетают со второй космической нахуй. Плутон сосет, сорри.
Аноним 05/03/21 Птн 19:13:37 639985501
>>639984
А если это не настолько мелкохуйня? Допустим, хотя бы с Луну или Меркурий?
Аноним 05/03/21 Птн 19:43:11 639990502
>>639985
Там все равно утекать будет. Всегда. Кинетическая энергия молекул газа позволяет им покидать гравитационное влияние. Вопрос лишь насколько быстро. Какая-нибудь холодная ледышка с почти жидкой атмосферой будет долго сохнуть.
Аноним 05/03/21 Птн 19:44:33 639992503
>>639990
А если захуячить в такую коротышку ядро и мантию из ирридия и осмия?
Аноним 05/03/21 Птн 19:46:46 639994504
>>639992
То будет еще дольше сохнуть, очевидно.
По идее даже посчитать несложно, сам возьми параметры своей планеты, тебе вроде надо только знать состав атмосферы, плотность, температуру и силу притяжения.
Иридий пишется через одну "р".
Аноним 05/03/21 Птн 20:28:07 640005505
16127697552360.webm 20461Кб, 640x360, 00:12:04
640x360
>>639994
Дык я не знаю. Я просто хочу понять, где минимальный предел комфортной для человека микропланетки. В голове мысли только о защитных магнитных полях от металлов в ядре и разогревом, индуциированным лунами/астероидами-лунами, а также толсто-налитая атмосфера (типа как на Титане, можно с добавлением крпитонов-аргонов всяких), дающие в сумме жидкую воду в далёких холодных ебенях местной солнечной системы. Ну а вес для гравитации только тяжметом добавлять.
Аноним 05/03/21 Птн 20:29:11 640006506
16127697552360.webm 20461Кб, 640x360, 00:12:04
640x360
>>639994
Дык я не знаю. Я просто хочу понять, где минимальный предел комфортной для человека микропланетки. В голове мысли только о защитных магнитных полях от металлов в ядре и разогревом, индуциированным лунами/астероидами-лунами, а также толсто-налитая атмосфера (типа как на Титане, можно с добавлением крпитонов-аргонов всяких), дающие в сумме жидкую воду в далёких холодных ебенях местной солнечной системы. Ну а вес для гравитации только тяжметом добавлять.
Аноним 05/03/21 Птн 21:01:45 640009507
>>640005
После какого-нибудь столкновения с выбросом волатильных говен и заново-разогрева планетки вполне можно получить жизнеспособную хуитку на миллионы лет, с постоянно утекающей атмосферой и остывающей жижей. Правда, после столкновения тоже ждать надо тысячи лет, чтоб все подутряслось.
Аноним 05/03/21 Птн 21:05:55 640010508
w800[1] 24Кб, 600x380
600x380
Почему считается что орбита гипотетической Планеты Х обязана пролегать 146% зуб даю где-то по красной траектории?
Почему не по зелёным, например?
Почему вообще считается, что раз она есть, обязательно вытянутая?
Аноним 05/03/21 Птн 22:23:29 640026509
>>640010
Вкратции - по орбитам долгопериодических комет, и объектов пояса Койпера, дохуя данных по ним проанализировали и смекнули что наличие массивной хуйни на такой орбите объясняет эти вещи.
Вподробнии читай статью на вике: https://en.wikipedia.org/wiki/Planet_Nine - там вся история с подробностями.
Аноним 06/03/21 Суб 02:16:33 640062510
Аноним 06/03/21 Суб 02:43:17 640065511
Чому никто тред не перекатывает?
Аноним 06/03/21 Суб 03:56:42 640070512
Аноним 09/03/21 Втр 11:52:50 640624513
>>640010
>Почему не по зелёным, например?
Потому что по зеленым пролетают другие тела
Настройки X
Ответить в тред X
15000
Макс объем: 40Mб, макс кол-во файлов: 4
Кликни/брось файл/ctrl-v
Стикеры X
Избранное / Топ тредов